OnlyIASPT292023 (Upscmaterial - Online)

You might also like

Download as pdf or txt
Download as pdf or txt
You are on page 1of 50

IDMP TEST SERIES-2023

High Yielding PRELIMS TEST SERIES

Test-29
International Relations/Important key
terminologies
Topic Covered:
 Important key terminologies, committees, reports in news
 International Relations

IDMP-2023
IDMP TEST SERIES-2023
High Yielding PRELIMS TEST SERIES

Test-29
अतं रार्��ीय सबं धं
Topic Covered:
 भारत म� बाहर� �ेत्र
 भारत म� सावर्ज�नक �व� और कराधान
 महत्वपण
ू र् अंतरार्ष्ट्र�य आ�थर्क संगठन, महत्वपूणर् भारतीय आ�थर्क �नकाय, अ�ध�नयम
 �रपोटर्, सूचकांक आ�द
 गर�बी और बेरोजगार�

IDMP-2023
Contact us :info@onlyias.com

OnlyIAS Nothing Else Visit :dpp.onlyias.in


Contact : +91-7007 931 912

Q.1) Which of the following statements about UN- Q.5) Which of the following statements about Eco
Habitat is/are correct? Sensitive Zones is/are correct?
1) It is a global program for conservation of habitat of 1) They are declared by Ministry of environment and
wild animals in their own habitat climate change under the wildlife protection act
2) The nodal agency for UN-Habitat in India is the 2) These zones would act as a transition zone from
Ministry of Housing and Urban Affairs areas of high protection to those involving lesser
Select the correct answer from the codes given below: protection.
a) 1 only 3) Ongoing agricultural or horticultural practices,
b) 2 only rainwater harvesting, organic farming these
c) 1 and 2 both activities are permitted in ESZ.
d) None of the above Select the correct answer from the codes given below:
a) 1 and 2 only
Q.2) Consider the following statements about Global b) 1 and 3 only
Environmental Facility(GEF) and identify the incorrect c) 2 and 3 only
ones: d) All of the above
1) It was set up as a fund under UNFCCC.
2) GEF serves as a financial mechanism for the Q.6) Which of the following statements about
Minamata Convention on Mercury. Community Forest Resource Rights under Forest
3) India has been a recipient but not donated to the Rights Act, is/are correct?
fund yet. 1) CFR rights, along with Community Rights (CRs)
Select the correct answer from the codes given below: include nistar rights which ensure sustainable
a) 1 and 3 only livelihoods of the community.
b) 1 and 2 only 2) These rights give the authority to the Gram Sabha
c) 2 and 3 only to adopt local traditional practices of forest
d) None of the above conservation.
Select the correct answer from the codes given below:
Q.3) Kunming Declaration which was in news a) 1 only
recently, is related to b) 2 only
a) Counter global terrorism c) 1 and 2 both
b) Refugee crisis d) None of the above
c) Biodiversity
d) Data security Q.7) SHRESHTA scheme was launched by
a) Ministry of culture
Q.4) Which of the following statements is/are correct b) Ministry of home affairs
about Kuno National Park? c) Ministry of social justice and empowerment
1) Wildlife Institute of India and Wildlife Trust of India d) Ministry of skill development
had shortlisted Palpur-Kuno park as a habitat for
Cheetahs and Asiatic lions. Q.8) Consider the following statements about Fishing
2) Kuno National Park is in Gujrat. Cats:
Select the correct answer from the codes given below: 1) They are found along the Eastern Ghats, tidal
a) 1 only mangrove forests and also inland freshwater
b) 2 only habitats.
c) Both 1 and 2 2) Its IUCN status is Endangered.
d) Neither 1 nor 2 Which of the above statement/s is/are incorrect?
a) 1 only

DPP 2023 DAY 202 1


Contact us :info@onlyias.com

OnlyIAS Nothing Else Visit :dpp.onlyias.in


Contact : +91-7007 931 912

b) 2 only Q.13) Which of the following statements about


c) 1 and 2 both Rashtriya Gram Swaraj Abhiyan is/are correct?
d) None of the above 1) Ministry of rural development is the implementing
agency for this program
Q.9) Which of the following statements about Sant 2) It envisaged developing governance capabilities of
Tukaram is/are correct? Panchayati Raj Institutions (PRIs) to deliver on the
1) Sant Tukaram was a Warkari saint and poet. Sustainable Development Goals (SDGs).
2) Tukaram’s teachings were regarded as Adviat- Select the correct answer from the codes given below:
based. a) 1 only
Select the correct answer from the codes given below: b) 2 only
a) 1 only c) 1 and 2 both
b) 2 only d) None of the above
c) 1 and 2 both
d) None of the above Q.14) Which of the following statements about the
Mid Day Meal scheme is/are correct?
Q.10) Project NIPUN is related to 1) Ministry of social justice is the nodal ministry for
a) Create a future labour force for the construction the program
industry. 2) The basic objective of this scheme is to enhance
b) Improve the digital literacy rate of senior citizens. enrolment in schools.
c) Develop local art and craft. Select the correct answer from the codes given below:
d) Provide capital access to distant locations of NE a) 1 only
states. b) 2 only
c) 1 and 2 both
Q.11) Which of the following statements about Keibul d) None of the above
Lamjao National Park is/are correct?
1) It is the only floating National Park in the world. Q.15) Consider the following statements about
2) It is located on the Loktak Lake which is the largest Ayushman Bharat Digital Mission:
freshwater lake in India. 1) The Department of Pharmaceuticals will be the
Select the correct answer from the codes given below: nodal ministry to implement this program.
a) 1 only 2) The Sandbox, created as a part of the mission, will
b) 2 only act as a framework for technology and product
c) 1 and 2 both testing.
d) None of the above Which of the statements given above is/are correct ?
a) 1 only
Q.12) Which of the following statement/s is/are b) 2 only
correct about Brow Antlered Deer? c) 1 and 2 both
1) It is the state animal of Arunachal pradesh. d) None of the above
2) Its IUCN status is Endangered.
Select the correct answer from the codes given below: Q.16) Which of the following statements is/are
a) 1 only correct about Pradhan Mantri Bhartiya Janaushadhi
b) 2 only Pariyojana (PMBJP) ?
c) 1 and 2 both 1) PMBJP is a campaign launched by the Department
d) None of the above of Pharmaceuticals.

DPP 2023 DAY 202 2


Download From - https://upscmaterial.online/

Contact us :info@onlyias.com

OnlyIAS Nothing Else Visit :dpp.onlyias.in


Contact : +91-7007 931 912

2) One of the objectives of this campaign is to Q.20) Theyyam was recently in news, it is
popularise generic medicines among the masses, a) One of the famous handicraft art of karnataka
because generic medicines are branded one. b) Martial art of kerala
Select the correct answer from the codes given below: c) Ritual form of dance worship in Kerala and
a) 1 only Karnataka
b) 2 only d) Sculptural technique
c) 1 and 2 both
d) None of the above Q.21) Which of the following statements about
Savitribai Phule is/are correct?
Q.17) Which of the following statements about 1) She worked to abolish the discrimination and
Minimum Assured Return Scheme (MARS) is/are unfair treatment of people based on caste and
correct? gender.
1) It offers a guaranteed minimum rate of return to 2) Savitribai started the Mahila Seva Mandal.
NPS (National Pension System) subscribers. Select the correct answer from the codes given below:
2) This scheme was launched by the insurance a) 1 only
regulatory and development authority. b) 2 only
Select the correct answer from the codes given below: c) 1 and 2 both
a) 1 only d) None of the above
b) 2 only
c) 1 and 2 both Q.22) Which of the following statements about
d) None of the above Glycosmis Albicarpa are correct?

e
1) The species belongs to the Orange family,
Q.18) About Private Member Bill, which of the

n
Rutaceae.
following statements is/are correct? 2) Berries of Glycosmis species have the unique

li
1) Responsibility to draft such bills lies with a member characteristic of ‘gin aroma’

n
who presents it. 3) The species is endemic to himalayan forests
2) Its rejection by the House has implications on
o
Select the correct answer from the codes given below:
l.
parliamentary confidence. a) 1 and 2 only
a

Select the correct answer from the codes given below: b) 1 and 3 only
a) 1 only c) 2 and 3 only
ri

b) 2 only d) All of the above


te

c) 1 and 2 both
d) None of the above Q.23) Consider the following statements about Start-
a

up Village Entrepreneurship Programme (SVEP):


m

Q.19) Which of the following statements about the 1) It aims at providing self-employment opportunities
Financial Action Task Force are correct? with financial assistance
c

1) It works for combating money laundering and 2) It promotes both individual and group enterprises
s

terrorism. 3) It was launched as a sub-scheme under skill india


p

2) Its secretariat is located at the United Nations. mission by ministry of skill development
.u

3) India is a member of it. Which of the statements given above is/are correct ?
Select the correct answer from the codes given below: a) 1 and 2 only
w

a) 1 and 2 only b) 1 and 3 only


b) 1 and 3 only c) 2 and 3 only
w

c) 2 and 3 only d) All of the above


w

d) All of the above

DPP 2023 DAY 202 3


https://upscmaterial.online/
Download From - https://upscmaterial.online/

Contact us :info@onlyias.com

OnlyIAS Nothing Else Visit :dpp.onlyias.in


Contact : +91-7007 931 912

Q.24) Which of the following statements is/are a) Ministry of industry and commerce
correct about National Supercomputing Mission: b) Federation of Indian Chambers of Commerce &
1. It supports the government's vision of 'Digital India' Industry
and 'Make in India' initiatives. c) Indian Institute of Foreign Trade
2. The Mission is being jointly steered by the d) NITI Aayog
Department of Science and Technology (DST) and
the ministry of skill development and Q.28) About the National Innovation Foundation,
entrepreneurship. which of the following statements is/are correct?
Select the correct answer from the codes given below: 1) It is an autonomous body set up by Department for
a) 1 only Promotion of Industry and Internal Trade
b) 2 only 2) It helps grassroot innovators and outstanding
c) 1 and 2 both traditional knowledge holders get due recognition,
d) None of the above respect and reward for their innovations
Select the correct answer from the codes given below:
Q.25) Which of the following statements is/are a) 1 only
correct about National Land Monetization b) 2 only
Corporation (NLMC) ? c) 1 and 2 both
1) The NLMC will undertake the monetization of d) None of the above
surplus land and building assets of Central Public
Sector Enterprises (CPSEs) and other government Q.29) Which of the following statements about the
bodies/organisations. United Nations Population Fund is/are correct?

e
2) NLMC will act as a repository of best practices in 1. It is a subsidiary organ of the UNESCO

n
land monetization, assist and provide technical 2. UNFPA is the world's largest multilateral source of
advice to Government in implementation of asset funding for population and reproductive health

li
monetization programme. programs.

n
Select the correct answer from the codes given below: Select the correct answer from the codes given below:
o
a) 1 only a) 1 only
l.
b) 2 only b) 2 only
a

c) 1 and 2 both c) 1 and 2 both


d) None of the above d) None of the above
ri
te

Q.26) Which of the following statements about Q.30) State Energy and Climate Index (SECI) was
Narmada River are correct? released by
a

1) Narmada is the largest east flowing river of the a) Ministry of power


m

peninsular region b) The Energy and Resources Institute


2) Hiran and Orsang are two tributaries among others c) NITI Aayog
c

3) It rises from Maikala range near Amarkantak in d) Ministry of environment and climate change
s

Madhya Pradesh.
p

Select the correct answer from the codes given below: Q.31) Which of the following are benefits of a Cashless
.u

a) 1 and 2 only Economy?


b) 1 and 3 only 1. Reduces the tax-burden and increased overall tax
w

c) 2 and 3 only revenue


d) All of the above
w

2. Reduces instances of black economy, tax


avoidance, and money laundering.
w

Q.27) Export Preparedness Index was released by 3. Reduce power consumption

DPP 2023 DAY 202 4


https://upscmaterial.online/
Download From - https://upscmaterial.online/

Contact us :info@onlyias.com

OnlyIAS Nothing Else Visit :dpp.onlyias.in


Contact : +91-7007 931 912

4. Inclusive growth and distributive social equity. d) None of the above


5. Promote Digital Equality
Choose the correct answer using code given below: Q.35) Recently, the Securities and Exchange Board of
a) 1, 2, 5 only India (SEBI) allowed stock exchanges to start the T+1
b) 1, 2, 4, 5 only system as an option in place of T+2 for the completion
c) 1, 2, 4 only of share transactions. Which of the following
d) 1, 2, 3, 4 and 5 statement best defines the T+2 system?
a) It means settlements will have to be cleared within
Q.32) India and the majority of the members of OECD- two day after the actual transaction takes place.
G20 Inclusive Framework on Base Erosion and Profit b) It means settlements will have to be cleared within
Shifting (BEPS) have joined a new two-pillar plan. The one day after the actual transaction takes place.
twin - pillar plan is related to which of the following ? c) It means settlements will have to be cleared within
a) The rules of trade between nations two week after the actual transaction takes place.
b) Trade-Related Aspects of Intellectual Property d) It means settlements will have to be cleared within
Rights two month after the actual transaction takes place.
c) Foreign direct investment (FDI)
d) Reformation of international taxation rules. Q.36) Consider the following statements :
1. The Interest Coverage Ratio (ICR) is a financial ratio
Q.33) Recently seen in news ‘SAMADHAAN Portal’ is that is used to determine how well a company can
associated with? pay the interest on its outstanding debts.
a) To help in monitoring the implementation of public 2. Firms with an interest coverage ratio higher than

e
procurement policy for micro and small one are unable to meet their interest obligations

n
enterprises. from their income , are categorised as zombies.
b) To ensure transparency in admissions, prevent

li
Which of the statements given above is/ are not
unfair practices in higher educational institutions correct ?

n
c) For empowering micro and small entrepreneurs a) 1 only o
across the country to directly register their cases b) 2 only
l.
relating to delayed payments. c) Both 1 and 2
a

d) A digital platform wherein jobseekers (passed out d) Neither 1 nor 2


ri

trainees/students of MSME Technology Centres)


and recruiters get connected. Q.37) The Gini coefficient (Gini index or Gini ratio) is a
te

statistical measure used to estimate which of the


Q.34) Consider the following recently formed advisory following?
a

Committees and their heads: a) Poverty level


m

1. Advisory committee on leveraging regulatory and b) Food Inflation


technology solutions (ALeRTS) : Sunil Bajpai c) Income distribution
c

2. Advisory committee for advising on ESG d) Unemployment


s

(environment, social and governance) related


p

matters in the securities market : Navneet Munot Q.38) Consider the following statements regarding
.u

3. Advisory committee on mutual funds : Usha Thorat Card Tokenization:


How many of the above pair is/are matched 1. It refers to replacement of actual credit and debit
w

correctly? card details with an alternate code called the “token”,


w

a) Only one pair which allows purchases to go through without


b) Only two pairs exposing sensitive details.
w

c) All three pairs

DPP 2023 DAY 202 5


https://upscmaterial.online/
Download From - https://upscmaterial.online/

Contact us :info@onlyias.com

OnlyIAS Nothing Else Visit :dpp.onlyias.in


Contact : +91-7007 931 912

2. It can be used for domestic and international d) Non performing assets (NPA’s) of public sector
transactions with a small fee. banks
Which of the statements given above is/are Incorrect?
a) 1 only Q.42) Which of the following statement is/are
b) 2 only incorrect with respect to Barbell Strategy?
c) Both 1 and 2 a) It combines a bouquet of safety-nets to cushion the
d) Neither 1 nor 2 impact on vulnerable sections of society/business.
b) It is an approach to uncertainty (risk) that uses two
Q.39) Consider the following statements in context of extremes with one end of the barbell being
e-RUPI: extreme risk aversion (safety),on the other end is
1. It is a cashless and contactless digital payment extreme risk loving (speculation).
medium, which will be delivered to mobile phones of c) It is a method that consists of taking both a
beneficiaries in the form of an SMS string or a QR code. defensive attitude and an excessively aggressive
2. e-RUPI does not require the beneficiary to have a one at the same time.
bank account and is operable on basic phones. d) None of the above
3. It is a Kind of digital voucher which is neither person-
specific nor purpose-specific and can be used by Q.43) Which of the following statement is Incorrect
individuals, corporates or governments. with respect to Refined core inflation?
Which of the statements given above is/are correct? a) It excludes main fuel items like ‘petrol for vehicle’,
a) 1 and 2 only ‘diesel for vehicle’ and ‘lubricants’ and other fuels
b) 2 only for vehicles, in addition to ‘food and beverages’

e
c) 1 and 3 only and ‘fuel and light’ from the headline retail

n
d) All of the above inflation.

li
b) It indicates the impact of inflation in fuel items in
Q.40) What are the benefits of International Trade the conventional core inflation measure.

n
settlements in Rupees: c) Refined core inflation is much higher than
o
1. Easing trade and better relations with other conventional core inflation.
l.
countries like Sri Lanka and Russia d) None of the above
a

2. Save foreign currency and reduce trade deficit


ri

3. Decrease India's exports and increase the risk of Q.44) Consider the following statements regarding
forex fluctuations. GUAREX :
te

4. Slow the depreciation of the rupee. 1. Guarex is an index tracking the price movement in
Which of the options given above is/are correct? the futures contracts of its underlying commodities
a

a) 1 and 3 only such as guarseed and guar gum refined splits on a real-
m

b) 2 and 4 only time basis.


c

c) 1, 2 and 4 only 2. It is launched by Multi commodity exchange (MCX).


d) All of the above 3. India is the world’s second largest guar producer.
s

Which of the statements given above is/are Incorrect?


p

Q.41) The Sudarshan Sen Committee formed by RBI a) 1 and 2 only


.u

for the review of? b) 2 and 3 only


a) Existing Market Infrastructure Institutions (MIIs) c) 3 only
w

b) Non Banking Financial Companies (NBFC) d) None of the above


w

c) Working of Asset Reconstruction Companies


(ARCs) Q.45) Which of the following will be possible results
w

of adopting Accommodative monetary policy?

DPP 2023 DAY 202 6


https://upscmaterial.online/
Download From - https://upscmaterial.online/

Contact us :info@onlyias.com

OnlyIAS Nothing Else Visit :dpp.onlyias.in


Contact : +91-7007 931 912

1. Decrease in money supply 4. ASEAN is India’s 2nd largest trading partner.


2. Encourage more spending from consumers and Select the correct answer using the code given below:
businesses a) 1, 2 and 3 only
3. lowering of short-term interest rates b) 3 and 4 only
4. Unemployment rate will increase. c) 2 and 3 only
Which of the options given above is/are correct: d) 1 and 4 only
a) 1 and 4 only
b) 2 and 3 only Q.49) Consider the following statements about
c) 1, 2 and 4 only Shanghai Cooperation Organization (SCO):
d) All of the above 1. SCO is a political, economic and military
organization aiming to maintain peace, security
Q.46) Consider the following statements in context of and stability in the region.
Liquidity Adjustment Facility (LaF) Corridor : 2. SCO was found in 2001 and India was the founding
1. It indicates the difference between the Repo rate member of SCO
and marginal standing facility (MSF) rate. 3. The city of Varanasi has been nominated as the
2. It is a tool used in monetary policy, which enables first-ever SCO Tourism and Cultural Capital during
banks to borrow money through repurchase the period 2022-2023 at the 22nd Meeting of
agreements or banks to lend to the RBI using reverse Shanghai Cooperation Organization (SCO) Council
repo contracts. of Heads of State.
Which of the statements given above is/are Incorrect? Which of the statements given above are correct ?
a) 1 only a) 1 and 2 only

e
b) 2 only b) 2 and 3 only

n
c) Both 1 and 2 c) 1 and 3 only

li
d) Neither 1 nor 2 d) All of the above

n
Q.47) With reference to REvil which was in the news o
recently, what is correct? Q.50) Consider the following statements regarding
l.
a) It was a Russia based private ransomware-as-a- Kazakhstan:
a

service (RaaS) operation. 1. Kazakhstan is the world’s second largest


ri

b) It was Israeli defence missile. landlocked country in the world.


c) It was a Chinese bio-weapon. 2. It is the top global producer of uranium.
te

d) It was USA’s drone spying global military 3. Recently there was unrest in Kazakhstan due to
operations to gather intelligence. rising fuel prices.
a

Which of the statements given above are correct ?


m

Q.48) Which of the following statements about a) 1 and 2 only


c

Association of Southeast Asian Nations (ASEAN) are b) 1 and 3 only


correct? c) 2 and 3 only
s

1. Association of Southeast Asian Nations (ASEAN) d) All of the above


p

was established as per the Hong-kong declaration


.u

in 1967. Q.51) Red Corner Notice was recently in news, which


2. India is also a part of ASEAN Defence Ministers of the following organizations issues this notice?
w

Meeting (ADMM)-Plus Meeting. a) International court of justice (ICJ)


w

3. The year 2022 marks 30 years of ASEAN-India b) United Nations Security Council (UNSC)
relations and it has been designated as ASEAN- c) INTERPOL
w

India Friendship Year d) Financial Action Task Force (FATF)

DPP 2023 DAY 202 7


https://upscmaterial.online/
Download From - https://upscmaterial.online/

Contact us :info@onlyias.com

OnlyIAS Nothing Else Visit :dpp.onlyias.in


Contact : +91-7007 931 912

Q.52) Consider the following statements regarding Which of the statements given above is/ are correct ?
UN Peacekeeping force : a) 1 only
1. The UN Peacekeepers are known as ‘Blue Helmets’ b) 2 only
or ‘Blue Berets’ and its personnel consist of c) Both 1 and 2
soldiers, military officers, police officers and d) Neither 1 nor 2
civilian personnel from many countries.
2. The UN Peacekeeping force is supplemented by
personnel from member nations which are added Q.55) Consider the following statements regarding
to the force on a mandatory basis, as per UN the Organisation of the Petroleum Exporting
charter . Countries OPEC :
Which of the statements given above is /are not 1. OPEC is a permanent, Non-governmental
correct ? organization, headquartered in Vienna,Austria.
a) 1 only 2. OPEC membership is open to any country that is a
b) 2 only substantial exporter of oil and which shares the
c) Both 1 and 2 ideals of the organization.
d) Neither 1 nor 2 3. The OPEC countries which export largest crude oil
are termed as OPEC plus countries.
Which of the statements given above is /are correct ?
Q.53) Consider the following statements regarding a) 2 and 3 only
the Association of Southeast Asian Nations: b) 2 only
1. ASEAN was established in 1967 in Indonesia with c) 1 and 3 only

e
the signing of the Manila Declaration d) 1, 2 and 3

n
2. ASEAN Plus Three is the consultative group

li
initiated in 1997 that brings together ASEAN’s ten
members, and India , Australia and New Zealand. Q.56) The Common Framework for debt treatment

n
3. The year 2022 marks the 10th anniversary of beyond the DSSI (Common Framework) is an initiative
o
India’s Strategic Partnership with ASEAN and has of the G20, together with the Paris Club intended to
l.
been designated as the ASEAN-India Friendship achieve which of the following ?
a

Year. a) To support Low-Income Countries with


ri

Which of the statements given above is /are correct ? unsustainable debt in a structural manner.
a) 2 only b) To help Pacific Islands often described as highly
te

b) 3 only vulnerable to climate change and lacking


c) 1 and 3 only adaptation option
a

d) 1, 2 and 3 c) To provide defence trade-related technical


m

assistance to African countries


c

Q.54) Consider the following statements regarding d) To support people vulnerable to food insecurity
the New Development Bank (NDB) : through humanitarian assistance
s

1. The New Development Bank (NDB) is a Multilateral


p

Development Institution (MDI) established by


.u

BRICS countries during the 4 th BRICS Summit in Q.57) Consider the following statements regarding
Delhi (2012). the G20 nations :
w

2. It has a weighted voting system where votes are 1. The G20 was formed in 2009 in the backdrop of the
w

distributed in proportion with members’ capital global financial crisis that hit the global economies,
subscriptions in which China is the largest especially the developed nations.
w

shareholder followed by India and Russia

DPP 2023 DAY 202 8


https://upscmaterial.online/
Download From - https://upscmaterial.online/

Contact us :info@onlyias.com

OnlyIAS Nothing Else Visit :dpp.onlyias.in


Contact : +91-7007 931 912

2. The permanent secretariat of G20 is located in Which of the statements given above is /are correct ?
Paris and the primary mandate of the grouping is a) 1 only
for International Economic cooperation with b) 2 and 3 only
particular emphasis to prevent future financial c) 3 only
crises across the world. d) 1, 2 and 3
Which of the statements given above is /are not
correct ?
a) 1 only Q.60) Both India and Myanmar are members of which
b) 2 only of the following organisations ?
c) Both 1 and 2 1. ASEAN (Association of Southeast Asian Nations)
d) Neither 1 nor 2 2. Bay of Bengal Initiative for Multi-Sectoral Technical
and Economic Cooperation (BIMSTEC)
3. South Asian Association for Regional Cooperation
Q.58) Consider the following statements regarding (SAARC)
Asian Infrastructure Investment Bank : 4. East Asia summit
1. Asian Infrastructure Investment Bank (AIIB) is a Select the correct answer using the codes given below
multilateral development bank with a mission to :
improve social and economic outcomes in Asia and a) 1 and 2 only
beyond. b) 2 and 4 only
2. USA is the largest shareholder with 26.61 % voting c) 1 and 3 only
shares in the bank followed by China and Japan d) 1, 2, 3 and 4

e
3. As a multilateral development bank focused on

n
developing Asia , AIIB does not have members Q.61) Consider the following statements regarding

li
outside Asia the Missile Technology Control Regime :
Which of the statements given above is/ are correct? 1. It is a binding agreement created by the major

n
a) 2 only supplier countries that have agreed to co-operate
o
b) 2 and 3 only in their effort to prevent and regulate the transfer
l.
c) 1 only of certain military and dual use technology.
a

d) 1, 2 and 3 2. The members are thus prohibited from supplying


ri

such missiles and UAV systems that are controlled


by the MTCR to other members of MTCR
te

Q.59) Consider the following statements regarding 3. India was inducted into the Missile Technology
International Criminal Police Organization Control Regime in 2016 as its member.
a

(INTERPOL): Which of the statements given above are not correct?


m

1. It was set up in 1923, as a secure information- a) 1 and 2 only


c

sharing platform that facilitates criminal b) 2 and 3 only


investigation of police forces across the globe that c) 1 and 3 only
s

is headquartered in Lyon, France. d) 1, 2 and 3


p

2. The National Security Council assumes this role in


.u

India with one of its senior officers heading its


exclusive wing for collation of information and Q.62) Consider the following statements regarding
w

liaison with the world body. the International Telecommunication Union (ITU):
w

3. The General Assembly of the International Criminal 1. It is an organisation outside the purview of the
Police Organisation (Interpol) held its meeting in United Nations whose purpose is to coordinate
w

Delhi for the first time in 2022.

DPP 2023 DAY 202 9


https://upscmaterial.online/
Download From - https://upscmaterial.online/

Contact us :info@onlyias.com

OnlyIAS Nothing Else Visit :dpp.onlyias.in


Contact : +91-7007 931 912

telecommunication operations and services session to address human rights violations and
throughout the world. emergencies.
2. India has been an active member of the ITU and has Which of the statements given above is /are correct?
been a regular member of the ITU Council a) 1 and 2 only
3. Membership of ITU is open to UN members, which b) 3 only
may join the Union as Member States, as well as to c) 1 and 3 only
private organisation. d) 1, 2 and 3
Which of the statements given above is /are correct?
a) 1 and 2 only Q.66) India has signed and ratified which of the
b) 2 and 3 only following Treaties/Conventions/ Codes ?
c) 1 and 3 only 1. Comprehensive Nuclear-Test-Ban Treaty, 1996
d) 1, 2 and 3 2. Treaty on the Non-Proliferation of Nuclear
Weapons (NPT), 1970
3. Chemical Weapons Convention (CWC),1992
Q.63) Geneva convention often seen in news is Select the correct answer using the code given below
regarding which of the following issues ? :
a) The Protection of Children and Co-operation in a) 1 and 2 only
Respect of Intercountry Adoption b) 2 only
b) The International Convention for the Prevention of c) 3 only
Pollution From Ships d) 1, 2 and 3
c) The standards for ethical treatment in war and

e
limiting the barbarity of war. Q.67) Consider the following statements regarding

n
d) An international agreement regarding the the Antarctic treaty :

li
recognition of mediated settlements. 1. The Antarctic Treaty of 1959 makes it mandatory
for all the member countries to have provisions to

n
Q.64) Minsk agreement , related to peace seeking prevent or check unlawful activities at their
o
settlement signed between which of the following research stations
l.
countries ? 2. Currently the Antarctic treaty has 54 parties but
a

a) Israel and Palestine India has never become a member of this treaty as
ri

b) Armenia and Azerbaijan India has no research stations in Antarctica


c) Sudan and South Sudan 3. The Treaty only applies to ice shelves and land, not
te

d) Ukraine and Russia the sea.


Which of the statements given above is/ are correct?
a

a) 1 and 2 only
m

Q.65) Consider the following statements regarding b) 2 and 3 only


c

United Nations Human Rights Council : c) 1 and 3 only


1. The Council was created by the United Nations d) 1, 2 and 3
s

General Assembly in 2006 and it consists of all the


p

members of the United Nations General Assembly.


.u

2. It allows only organisations to bring human rights Q.68) “Davos in the desert” is the famous description
violations to the attention of the Council and not for which of the following ?
w

individuals a) Eastern Economic Forum


w

3. The UNHRC holds regular sessions three times a b) Future Investment Initiative
year and can decide at any time to hold a special c) Paris climate fund deal
w

DPP 2023 DAY 202 10


https://upscmaterial.online/
Download From - https://upscmaterial.online/

Contact us :info@onlyias.com

OnlyIAS Nothing Else Visit :dpp.onlyias.in


Contact : +91-7007 931 912

d) WEF’s initiative for investment in development of 3) India and Nepal share BBIN, BIMSTEC, NAM and
African countries affected by desertification SAARC
4) India is Nepal’s largest trade partner and the
Q.69) Consider the following statements about United largest source of foreign investments
Nation Security Council: Which of the statements given above are correct?
1) Its primary responsibility is to work to maintain a) 1, 2 and 3 only
international peace and security b) 2, 3 and 4 only
2) The council has 10 members, the five permanent c) 1, 2 and 4 only
members and five non-permanent members d) All of the above
elected for two-year terms.
3) India, for the eighth time, has entered the UNSC as Q.72) Which of the following statements regarding
a non-permanent member last year (2021) Indian Technical and Economic Cooperation
4) The five permanent members are the United Programme (ITEC) are correct?
States, the Russian Federation, Germany, China 1) It is a demand-driven, response-oriented
and the United Kingdom programme that focuses on addressing the needs
Which of the statements given above are correct ? of developing countries through innovative
a) 1 and 2 only technological cooperation between India and the
b) 3 and 4 only partnering nation.
c) 2 and 4 only 2) The ITEC Programme is fully funded by the
d) 1 and 3 only Government of India.
3) Division of Development Partnership

e
Q.70) Which of the following statements about the Administration (DPA) in the Ministry of External

n
Non-Proliferation Treaty (NPT) are correct? Affairs is the nodal division for handling all capacity

li
1) The NPT is an international treaty whose objective building programmes.
is to prevent the spread of nuclear weapons and Select the correct answer code from the following:

n
weapons technology, to foster the peaceful uses of a) 1 and 2 only o
nuclear energy, and to further the goal of b) 1 and 3 only
l.
disarmament. c) 2 and 3 only
a

2) Presently, it has 190 member states. India is a d) All of the above


ri

member of NPT.
3) It requires countries to give up any present or Q.73) Consider the following statements regarding
te

future plans to build nuclear weapons in return for TAPI Pipeline which has been in news constantly :
access to peaceful uses of nuclear energy. 1) TAPI Pipeline is a natural gas pipeline that
a

Select the correct code among the following : originates from Turkmenistan and passes through
m

a) 1 and 2 only Afghanistan and Pakistan to reach India.


c

b) 1 and 3 only 2) The TAPI project is being funded by the Asian


c) 2 and 3 only Development Bank (ADB).
s

d) All of the above 3) The TAPI pipeline is also called the Peace pipeline.
p

Which of the statements given above are correct?


.u

Q.71) With reference to India - Nepal relations, a) 1 and 2 only


consider the following statements: b) 2 and 3 only
w

1) Nepal being a landlocked country, it is surrounded c) 1 and 3 only


w

by India from three sides d) All of the above


2) India Nepal joint military exercise is known as Surya
w

Kiran

DPP 2023 DAY 202 11


https://upscmaterial.online/
Download From - https://upscmaterial.online/

Contact us :info@onlyias.com

OnlyIAS Nothing Else Visit :dpp.onlyias.in


Contact : +91-7007 931 912

Q.74) Which of the following correctly explains the 1. Ashgabat Agreement is a pipeline agreement for
meaning of De-hyphenated foreign policy? establishing the international pipeline to facilitate
a) In international politics, de-hyphenation means the exclusive transportation of Natural gas
dealing with two countries independently. between Central Asia and the Persian Gulf.
b) In international politics, de-hyphenation means 2. China, India ,Pakistan and all central Asian
dealing with two countries, having an adversarial
countries are members of Ashgabat agreement
relationship between them, in an independent
manner. Which of the statements given above is/ are correct?
c) In international politics, de-hyphenation means a) 1 only
dealing with two countries in such a manner where b) 2 only
strategic interest of both the countries is kept on c) Both 1 and 2
priority. d) Neither 1 nor 2
d) In international politics, de-hyphenation means
dealing with two countries who have adversarial
Q.78) Consider the following statements regarding
relations among them in such a way that relations
among adversarial countries normalize. Eastern Economic Forum :
1. EEF is an international forum organised each year
Q.75) Which of the following are correct about the to support the economic development of China’s
Organization for Security and Cooperation in Europe Far East and to expand international cooperation
(OSCE)? in the Asia-Pacific region
1) It deals with a wide range of security-related 2. This forum is sponsored by the organising
concerns, including arms control, nonproliferation, committee appointed by the Association of
counterterrorism, democratization, and human Southeast Asian Nations (ASEAN)

e
rights. Which of the statements given above is /are not
2) OSCE is the world’s largest regional security

n
correct ?
organization.

li
a) 1 only
3) India is among the 57 members of OSCE.
b) 2 only

n
Select the correct answer code from the following:
a) 1 and 2 only c) Both 1 and 2 o
b) 1 and 3 only d) Neither 1 nor 2
l.
c) 2 and 3 only
a

d) All of the above Q.79) Green Strategic Partnership, recently seen in


ri

news relates to which of the following countries?


Q.76) Which of the following are members of New a) USA and India
te

Quad grouping? b) France and India


1) USA c) Japan and China
a

2) Israel
d) Denmark and India
m

3) Japan
4) Australia
c

5) UAE Q.80) Consider the following statements:


6) India 1. The TDC fund has been recently launched by India
s

Select the correct answer code from the following: to involve private sectors with the support of the
p

a) 1, 3, 4 and 6 only state for investments in the African region


.u

b) 1, 2, 5 and 6 only 2. India and the United Kingdom have launched their
c) 2, 3, 5 and 6 only Global Innovation Partnership (GIP) as both
w

d) 2, 3, 4 and 5 only
countries look to bolster their innovation
w

ecosystem to benefit the start-ups.


Q.77) Consider the following statements regarding
Which of the statements given above is /are correct?
w

the Ashgabat Agreement :


a) 1 only

DPP 2023 DAY 202 12


https://upscmaterial.online/
Download From - https://upscmaterial.online/

Contact us :info@onlyias.com

OnlyIAS Nothing Else Visit :dpp.onlyias.in


Contact : +91-7007 931 912

b) 2 only Which of the statements given above is/are not


c) Both 1 and 2 correct ?
d) Neither 1 nor 2 a) 1 only
b) 2 only
Q.81) Consider the following statements regarding c) Both 1 and 2
Inter-Ministerial Coordination Group (IMCG): d) Neither 1 nor 2
1. The Inter-Ministerial Coordination Group (IMCG)
has been set up as a high-level mechanism towards Q.84) Consider the following statements regarding
mainstreaming of India's Act East' policy. I2U2 summit :
2. Recently, the first meeting of the Inter-Ministerial 1. I2U2 stands for India, Israel, the UK, and the US,
Coordination Group (IMCG) at Ministerial level was formed a new grouping to discuss common areas
convened by India’s Foreign Minister . of mutual interest, to strengthen the economic
Which of the statements given above is/ are correct ? partnership in trade and investment
a) 1 only 2. I2U2 was initially formed in 2021 following the
b) 2 only Abraham Accords, between Israel and Palestine is
c) Both 1 and 2 mediated by the USA.
d) Neither 1 nor 2 Which of the statements given above is /are correct ?
a) 1 only
Q.82) Consider the following statements regarding b) 2 only
Indian Ocean Naval Symposium : c) Both 1 and 2
1. It is a biennial event which was conceived by the d) Neither 1 nor 2

e
External affairs ministry , that brings together

n
navies of Indian Ocean Region littoral states to Q.85) About 'Domestic Institutional Investors (DIIs)'

li
increase maritime co-operation and enhance which of the following statements is/are correct?
regional security. 1) They are the investors which undertake

n
2. IONS includes 24 member nations that touch or lie investment in securities and other financial assets
o
within the Indian Ocean Region (IOR), and 8 of the country they are based in.
l.
observer nations. 2) Domestic institutional investors also affect the net
a

3. India assumed the Chairmanship of Indian Ocean investment flows into the economy
ri

Naval Symposium 2021 for a two-year tenure. Select the correct answer from the codes given below:
Which of the statements given above is /are correct? a) 1 only
te

a) 2 only b) 2 only
b) 3 only c) 1 and 2 both
a

c) 1 and 3 only d) None of the above


m

d) 1, 2 and 3
Q.86) Which of the following statements about
c

Q.83) Consider the following statements regarding 'Imperfect competition’ is/are correct?
s

One Country Two Systems approach : 1) Imperfect competition is a competitive market


p

1. The Hong Kong and Macau Special Administrative situation where there are only a few sellers selling
.u

Regions, both former colonies, Should have the heterogeneous goods.


same economic systems but can have different 2) Monopoly is an example of imperfect competition.
w

political systems as that of mainland China. Select the correct answer from the codes given below:
w

2. Their mini-Constitutions would remain valid for 50 a) 1 only


years till 2047 for Hong Kong and 2049 for Macau. b) 2 only
w

c) 1 and 2 both

DPP 2023 DAY 202 13


https://upscmaterial.online/
Download From - https://upscmaterial.online/

Contact us :info@onlyias.com

OnlyIAS Nothing Else Visit :dpp.onlyias.in


Contact : +91-7007 931 912

d) None of the above 1) Calibrated Tightening means during the current


rate cycle, a cut in the repo rate is off the table.
Q.87) What is 'Windfall Gains' ? 2) There can be an overall tilt towards a rate hike but
a) Expected gain in income of new start up not an increase in every policy meeting.
b) Unexpected gain in income due to winning a 3) Calibrated Tightening cannot happen outside the
lottery policy meetings.
c) Predicted gains in income based on audit Select the correct answer from the codes given below:
d) Unpredicted gains in income in weak market a) 1 and 2 only
situation b) 1 and 3 only
c) 2 and 3 only
d) All of the above
Q.88) 'Tulip Mania' is a metaphor used to describe?
a) Recession in food and beverages industry Q.92) The term Card Skimming is related to which of
b) Employment crisis the following?
c) Economic bubble a) It is the legal act of making credit or debit cards.
d) Exponential and sustainable growth b) When a customer loses a credit or debit card, the
bank sends a new card.
Q.89) About Black Swan Event, which of the following c) It is the illegal act of making unauthorised copies
statements is/are correct? of credit or debit cards.
1) A black swan is a predictable event that is beyond d) It is the removal of unused cards from the market.
what is normally unexpected of a situation.
2) Black swan events are characterised by their Q.93) Which of the following statements are correct

e
extreme rarity, severe impact, and the widespread about the 'Paradox of Thrift'?

n
insistence they were obvious in hindsight. 1) The paradox of thrift refers to a situation in which

li
Select the correct answer from the codes given below: people tend to spend more money, thereby
a) 1 only

n
leading to a fall in aggregate savings of the
b) 2 only
c) Both 1 and 2 economy as a whole. o
2) The paradox of thrift postulated by the British
l.
d) Neither 1 nor 2
Keynesian economist, John Maynard Keynes.
a

Q.90) Which of the following statements is/are 3) This paradox of thrift leads to a decrease in the
ri

correct about Hawkish Stance? level of employment and income and reduces total
te

1) In order to keep inflation in check, the Hawkish savings in the economy.


stance favours low-interest rates. Select the correct answer from the codes given below:
a

2) Hawkish Stance is generally not concerned with a) 1 and 2 only


b) 2 and 3 only
m

economic growth but supports an economy


operating at a level below its full-employment c) 1 and 3 only
c

equilibrium. d) All of the above


s

Select the correct answer from the codes given below:


p

a) 1 only Q.94) Which of the following statements is/are


b) 2 only incorrect about 'Catch up Effect'?
.u

c) Both 1 and 2 1) Catch up effect is alternatively called the theory of


d) Neither 1 nor 2
w

divergence.
2) Catch up effect states that poor or developing
Q.91) With reference to Calibrated Tightening, which
w

economies grow slower compared to developed


of the following statements are correct?
economies.
w

Select the correct answer from the codes given below:

DPP 2023 DAY 202 14


https://upscmaterial.online/
Download From - https://upscmaterial.online/

Contact us :info@onlyias.com

OnlyIAS Nothing Else Visit :dpp.onlyias.in


Contact : +91-7007 931 912

a) 1 only 1) Recessionary Gap is also termed as contractionary


b) 2 only gap.
c) Both 1 and 2 2) Recessionary Gap is a situation wherein the real
d) Neither 1 nor 2 GDP is higher than the potential GDP at the full
employment level.
Q.95) With reference to the 'Indifference Curve', Select the correct answer from the codes given below:
which of the following statements are correct? a) 1 only
1) An indifference curve is a graph showing a b) 2 only
combination of two goods that give the consumer c) Both 1 and 2
equal satisfaction and utility. d) Neither 1 nor 2
2) The indifference curve is drawn as an upward
sloping convex to the origin. Q.99) Which of the following statements about
3) Higher indifference curve represents a higher level 'Stimulus Package' is/are correct?
of satisfaction. 1. Stimulus package is a package of interest
Select the correct answer from the codes given below: subvention by the central banks of various
a) 1 and 2 only countries to stimulate the economy and save their
b) 1 and 3 only country from a financial crisis.
c) 2 and 3 only 2. The idea behind a stimulus package is to boost
d) All of the above spending, as spending increases demand.
Select the correct answer from the codes given below:
Q.96) The term Contagion in economy describes? a) 1 only

e
a) Spread of economic crisis or boom across countries b) 2 only
c) 1 and 2 both

n
or regions.
b) Isolationist tendency of economy. d) None of the above

li
c) Fluctuations in value of currency.

n
d) Economic impact on the political state of the Q.100) About the monetary policy committee, which
nation.
o
of the following statements is/are correct ?
l.
1) It is a policy by the finance ministry of countries
a

Q.97) Which of the following best describes Pareto related to tax.


2) Under the monetary policy committee 3 members
ri

efficiency?
a) It is a situation where no individual or preference are to be appointed by the central government.
te

criterion can be made better off without making at Select the correct answer from the codes given below:
least one individual or preference criterion worse a) 1 only
a

off. b) 2 only
m

b) It is a situation where all individuals can be made c) 1 and 2 both


better off but at the cost of few individuals. d) None of the above
c

c) It is a situation where all individuals can be made


s

better off without making any individual or


p

preference criterion worse off.


.u

d) It is a situation where few individuals can be made


better off without making at least one individual or
w

preference criterion worse off.


w

Q.98) Which of the following statements is/are


w

correct about the ‘Recessionary Gap'?

DPP 2023 DAY 202 15


https://upscmaterial.online/
Download From - https://upscmaterial.online/

Contact us :info@onlyias.com

OnlyIAS Nothing Else Visit :dpp.onlyias.in


Contact : +91-7007 931 912

Q.1) Ans: B ● Statement 3 is incorrect: India is both donor


Exp: and recipient of GEF.
● Statement 1 is incorrect: United Nations
Habitat Program (UN-Habitat) is a global
programme for Urban development that Q.3) Ans: C
promotes sustainable human settlements. The Exp:
United Nations Human Settlements ● Option C is correct: The Kunming Declaration
Programme, UN-HABITAT, is the United was adopted by over 100 countries at the
Nations agency for human settlements. It is ongoing 15th Conference of the Parties to the
mandated by the UN General Assembly to United Nations Convention on Biological
promote socially and environmentally Diversity in China. It calls for urgent and
sustainable towns and cities with the goal of integrated action to reflect biodiversity
providing adequate shelter for all. considerations in all sectors of the global
● Statement 2 is correct: India has been elected economy but crucial issues - like funding
to the Executive Board of the first UN-Habitat conservation in poorer countries and
Assembly at the Plenary Session of the committing to biodiversity-friendly supply
Assembly that was held in Nairobi in 2019. The chains have been left to discuss later. It is not a
nodal agency for UN-Habitat in India is the binding international agreement. It calls upon
Ministry of Housing and Urban Affairs. The UN the parties to mainstream biodiversity
body has an office in New Delhi and has been protection in decision-making and recognise
in the country since 1991. the importance of conservation in protecting

e
human health.

n
Q.2) Ans: A

li
Exp:
It is an independently operating financial organization. Q.4) Ans: A

n
● Statement 1 is incorrect: It was set up as a fund Exp: o
under the World Bank in 1991. In 1992, at the ● Statement 1 is correct: Wildlife Institute of
l.
Rio Earth Summit, the GEF was restructured India and Wildlife Trust of India had shortlisted
a

and moved out of the World Bank system to Palpur-Kuno park as a habitat for Cheetahs and
ri

become a permanent, separate institution. Asiatic lions.


● Statement 2 is correct: ● Statement 2 is incorrect: Kuno National Park is
te

○ GEF serves as a financial mechanism a national park in the Sheopur district of MP


for: established in 1981 as a wildlife sanctuary. In
a

■ Convention on Biological 2018, it was given the status of a national park.


m

Diversity (CBD) It is part of the Khathiar-Gir dry deciduous


c

■ United Nations Framework forests.


Convention on Climate
s

Change (UNFCCC)
p

■ UN Convention to Combat
.u

Desertification (UNCCD) Q.5) Ans: C


■ Stockholm Convention on Exp:
w

Persistent Organic Pollutants ● Statement 1 is incorrect: The National Wildlife


w

(POPs) Action Plan (2002-2016) of the Ministry of


■ Minamata Convention on Environment, Forest and Climate Change
w

Mercury (MoEFCC) stipulated that state governments

DPP 2023 DAY 202 16


https://upscmaterial.online/
Download From - https://upscmaterial.online/

Contact us :info@onlyias.com

OnlyIAS Nothing Else Visit :dpp.onlyias.in


Contact : +91-7007 931 912

should declare land falling within 10 km of the regenerate or conserve or manage” the community
boundaries of national parks and wildlife forest resource.
sanctuaries as eco fragile zones or Eco ● Statement 1 is correct: CFR rights, along with
Sensitive Zones (ESZs) under the Community Rights (CRs) under Sections 3(1)(b)
Environmental (Protection) Act, 1986. and 3(1)(c), which include nistar rights (used
● Statement 2 is correct: The purpose of erstwhile in princely states or zamindari, etc.)
declaring ESZs around national parks, forests and rights over non-timber forest products,
and sanctuaries is to create some kind of a ensure sustainable livelihoods of the
“shock absorber” for the protected areas. community.
These zones would act as a transition zone ● Statement 2 is correct: These rights give the
from areas of high protection to those authority to the Gram Sabha to adopt local
involving lesser protection. traditional practices of forest conservation and
● Statement 3 is correct: management within the community forest
resource boundary.
Prohibited Regulated Permitted
activities activities activities Q.7) Ans: C
Exp:
Commercial Felling of trees, Ongoing ● Option C is correct: The Ministry of Social
mining, saw mills, establishment agricultural or Justice and Empowerment has launched the
industries causing of hotels and horticultural
scheme ‘SHRESHTA.’ This scheme is known as
pollution, resorts, practices,
establishment of commercial use rainwater Scheme for residential education for students

e
major of natural harvesting, in High school in Targeted Areas. The scheme

n
hydroelectric water, erection organic farming, ‘SHRESHTA’ was created with the goal of
projects (HEP), of electrical use of

li
providing quality education and opportunity to
commercial use of cables, drastic renewable students of SC Category.

n
wood. change of energy sources,
agriculture adoption of o
Tourism activities system, e.g. green
l.
like hot-air adoption of technology for Q.8) Ans: B
a

balloons over the heavy all activities. Exp:


ri

National Park, technology, ● Statement 1 is correct: Fishing cats have a


discharge of pesticides etc, patchy distribution along the Eastern Ghats.
te

effluents or any widening of They abound in estuarine floodplains, tidal


solid waste or roads.
mangrove forests and also inland freshwater
a

production of
habitats. Apart from Sundarbans in West
m

hazardous
substances. Bengal and Bangladesh, fishing cats inhabit the
c

Chilika lagoon and surrounding wetlands in


Odisha, Coringa and Krishna mangroves in
s

Andhra Pradesh.
p

● Statement 2 is incorrect: IUCN status of fishing


.u

Q.6) Ans: C
Exp: cat is Vulnerable. Despite multiple threats, the
Fishing Cat was recently downlisted to
w

Under Section 3(1)(i) of the Scheduled Tribes and Other


Traditional Forest Dwellers (Recognition of Forest “Vulnerable” from “Endangered” in the IUCN
w

Rights) Act (commonly referred to as the Forest Rights Red List species assessment.
w

Act or the FRA), 2006 the Community Forest Resource


rights provide for recognition of the right to “protect,
DPP 2023 DAY 202 17
https://upscmaterial.online/
Download From - https://upscmaterial.online/

Contact us :info@onlyias.com

OnlyIAS Nothing Else Visit :dpp.onlyias.in


Contact : +91-7007 931 912

Q.9) Ans: A India, which is situated at Bandipora District of


Exp: Indian state Jammu & Kashmir.
● Statement 1 is correct: Sant Tukaram was a
Warkari saint and poet. The sect is spread Q.12) Ans: B
across Maharashtra and Sant Tukaram and his Exp:
works are central to it. He was famously known ● Statement 1 is incorrect: The brow-antlered
for Abhanga devotional poetry and deer, or sangai, is the state animal of Manipur.
community-oriented worship through spiritual In India, these animals are found only in
songs known as Kirtans. Manipur’s famed Loktak lake.
● Statement 2 is incorrect: Tukaram’s teachings ● Statement 2 is correct: Its IUCN status is
were regarded as Vedanta-based. Endangered. Sangai was believed to be almost
extinct by 1950, but six individuals were
Q.10) Ans: A spotted in 1953 and the State of Manipur has
Exp: protected the species to increase the
● Option A is correct: NIPUN’ i.e. National population to 204. Sangai faces threat from
Initiative for Promoting Upskilling of Nirman steadily degenerating habitat of phumdi as a
Workers result of continuous inundation and flooding
○ NIPUN is creating a future labour force caused due to artificial reservoir. Water quality
for the construction industry which of the reservoir is degrading due to pollution
will propel innovation and large-scale and stoppage of nutrient supply. There is also
development in the country. invasion of non-native plants like Paragrass.

e
○ The project NIPUN is an initiative of There has been decrease in area of phumdi

n
the Ministry of Housing & Urban from 31.60 km2 in 1993 to 23.72 km2 in 2010.

li
Affairs (MoHUA).
○ This project is running under the

n
flagship programme of the Deendayal Q.13) Ans: B o
Antyodaya Yojana-National Urban Exp:
l.
Livelihoods Mission (DAY-NULM) ● Statement 1 is incorrect: Ministry of
a

○ The basic motive of the project is to Panchayati Raj is the implementing agency of
ri

train over 1 lakh construction workers, this program. Scheme is aimed to develop
through fresh skilling and upskilling governance capabilities of Panchayati Raj
te

programmes. Institutions (PRIs).


● Statement 2 is correct: The main Central
a

Q.11) Ans: A Components were Incentivisation of


m

Exp: Panchayats and Mission Mode Project on e-


c

● Statement 1 is correct: It is the only floating Panchayat including other activities at Central
National Park in the world, the Keibul Lamjao level. It envisaged developing governance
s

National Park located on the Loktak Lake is the capabilities of Panchayati Raj Institutions (PRIs)
p

last natural habitat of the 'Sangai' (Rucervus to deliver on the Sustainable Development
.u

eldii eldii), the dancing deer of Manipur. Goals (SDGs).


● Statement 2 is incorrect: Loktak Lake is the
w

largest freshwater lake in Northeast India and Q.14) Ans: B


w

is famous for the phumdis floating over it. Exp:


Wular Lake is the largest freshwater lake in ● Statement 1 is incorrect: Ministry of Education
w

is the nodal agency for this program.

DPP 2023 DAY 202 18


https://upscmaterial.online/
Download From - https://upscmaterial.online/

Contact us :info@onlyias.com

OnlyIAS Nothing Else Visit :dpp.onlyias.in


Contact : +91-7007 931 912

Programme has been renamed as PM Poshan Undertakings (PSUs) of India works under the
Shakti Nirman or PM Poshan. It is not just a Ministry of Chemicals & Fertilisers.
scheme, but a legal entitlement of all school- ● Statement 2 is correct: One of the objectives
going children in primary and upper primary of PMBJP is to popularise generic medicines
classes, through the National Food Security Act among the masses and dispel the prevalent
(NFSA), 2013. notion that low priced generic medicines are of
● Statement 2 is correct: Mid Day Meal scheme inferior quality or are less effective. Generic
aims to enhance enrolment in schools. The medicines are unbranded medicines which are
MDM Scheme has many potential benefits: equally safe and have the same efficacy as that
attracting children from disadvantaged of branded medicines in terms of their
sections (especially girls, Dalits and Adivasis) to therapeutic value.
school, improving regularity, nutritional
benefits, socialisation benefits and benefits to Q.17) Ans: A
women are some that have been highlighted. Exp:
● Statement 1 is correct: This scheme offer a
Q.15) Ans: B guaranteed minimum rate of return to NPS
Exp: (National Pension System) subscribers,
● Statement 1 is incorrect: Ayushman Bharat especially those who are risk averse.
Digital Mission aims to provide digital health ● Statement 2 is incorrect: Minimum Assured
IDs for all Indian citizens to help hospitals, Return Scheme (MARS) was launched by The
insurance firms, and citizens access health Pension Fund Regulatory and Development

e
records electronically when required. The Authority (PFRDA). This will be the first scheme

n
National Health Authority (NHA) under the from the pension regulator that will offer a

li
Ministry of Health and Family Welfare will be guaranteed return to investors.
the implementing Agency.

n
● Statement 2 is correct: The Sandbox, created Q.18) Ans: A o
as a part of the mission, will act as a framework Exp:
l.
for technology and product testing that will ● Statement 1 is correct: Drafting of private
a

help organisations, including private players members bill is the responsibility of the
ri

intending to be a part of the national digital member concerned. Its introduction in the
health ecosystem become a Health House requires one month’s notice.
te

Information Provider or Health Information ● Statement 2 is incorrect: Its rejection by the


User or efficiently link with building blocks of House has no implication on the parliamentary
a

Ayushman Bharat Digital Mission. confidence in the government or its


m

resignation.
c

Q.16) Ans: C Q.19) Ans: B


s

Exp: Exp:
p

● Statement 1 is correct: PMBJP is a campaign ● Statement 1 is correct: Objective of FATF is to


.u

launched by the Department of set standards and promote effective


Pharmaceuticals in 2008 under the name Jan implementation of legal, regulatory and
w

Aushadhi Campaign. Bureau of Pharma PSUs of operational measures for combating money
w

India (BPPI) is the implementation agency for laundering, terrorist financing and other
PMBJP. The Bureau of Pharma Public Sector related threats to the integrity of the
w

international financial system.

DPP 2023 DAY 202 19


https://upscmaterial.online/
Download From - https://upscmaterial.online/

Contact us :info@onlyias.com

OnlyIAS Nothing Else Visit :dpp.onlyias.in


Contact : +91-7007 931 912

● Statement 2 is incorrect: Its Secretariat is ● Statement 1 is correct: A team of scientists


located at the Organisation for Economic from the Botanical Survey of India (BSI) has
Cooperation and Development (OECD) discovered a new gin berry species from the
headquarters in Paris. Kanyakumari Wildlife Sanctuary in Tamil Nadu.
● Statement 3 is correct: The FATF currently has The species belongs to the Orange family,
39 members including two regional Rutaceae.
organisations - the European Commission and ● Statement 2 is correct: Berries of Glycosmis
Gulf Cooperation Council. India is a member of species have the unique characteristic of ‘gin
the FATF. aroma’ and have gained in popularity as an
edible fruit. The species is also a larval host
Q.20) Ans: C plant for butterflies like other species of
Exp: Glycosmis
● Option C is correct: Theyyam is a popular ritual ● Statement 3 is incorrect: The species, named
form of dance worship in Kerala and Karnataka, Glycosmis albicarpa with a distinct large white
India. It consisted of thousand-year-old fruit, is endemic to the southern Western
traditions, rituals and customs. The people Ghats.
consider Theyyam itself as a channel to a god
and they thus seek blessings from Theyyam. Q.23) Ans: A
Each Theyyam is a man or a woman who Exp:
attained divine status by performing heroic ● Statement 1 is correct: Aim and objectives of
deeds or by leading a virtuous life. the scheme are:

e
1. Support the rural poor to come out of poverty.

n
Q.21) Ans: C 2. Providing self-employment opportunities with

li
Exp: financial assistance and training in business
● Statement 1 is correct: She is considered to be management and soft skills.

n
the pioneer of India's feminist movement. 3. Create local community cadres for promotion
o
Savitribai and her husband founded one of the of enterprises.
l.
first modern Indian girls' schools in Pune, at ● Statement 2 is correct: It promotes both
a

Bhide wada in 1848. She worked to abolish the individual and group enterprises, majorly in
ri

discrimination and unfair treatment of people manufacturing, trading and service sectors. It
based on caste and gender. She is regarded as invests on building the capacities of the
te

an important figure of the social reform entrepreneurs to run the businesses profitably
movement in Maharashtra. based on the local demand and ecosystem.
a

● Statement 2 is correct: In 1852, Savitribai ● Statement 3 is incorrect: SVEP is a sub-scheme


m

started the Mahila Seva Mandal to raise of the Deendayal Antyodaya Yojana-National
c

awareness about women’s rights. She Rural Livelihood Mission (DAY-NRLM), Ministry
simultaneously campaigned against child of Rural Development and has been
s

marriage, while supporting widow remarriage. implemented since 2016.


p

She initiated the first Satyashodhak marriage—


.u

a marriage without a dowry, Brahmin priests or Q.24) Ans: A


Brahminical rituals in 1873. Exp:
w

● Statement 1 is correct: It supports the


w

Q.22) Ans: A government's vision of 'Digital India' and 'Make


Exp: in India' initiatives. National Supercomputing
w

Mission was launched to enhance the research

DPP 2023 DAY 202 20


https://upscmaterial.online/
Download From - https://upscmaterial.online/

Contact us :info@onlyias.com

OnlyIAS Nothing Else Visit :dpp.onlyias.in


Contact : +91-7007 931 912

capacities and capabilities in the country by Exp:


connecting them to form a Supercomputing ● Statement 1 is incorrect: Narmada is the
grid, with National Knowledge Network (NKN) largest west flowing river of the peninsular
as the backbone. The NKN project is aimed at region flowing through a rift valley between
establishing a strong and robust Indian the Vindhya Range on the north and the
network which will be capable of providing Satpura Range on the south.
secure and reliable connectivity. ● Statement 2 is correct: Hiran, Orsang, the
● Statement 2 is incorrect: The Mission is being Barna and the Kolar are major tributaries of
jointly steered by the Department of Science Narmada river
and Technology (DST) and the Ministry of ● Statement 3 is correct: It rises from Maikala
Electronics and Information Technology range near Amarkantak in Madhya Pradesh. It
(MeitY). It is implemented by the Centre for drains a large area in Madhya Pradesh besides
Development of Advanced Computing (C- some areas in the states of Maharashtra and
DAC), Pune, and the IISc, Bengaluru. Gujarat.

Q.25) Ans: C Q.27) Ans: D


Exp: Exp:
● Statement 1 is correct: The NLMC will ● Option D is correct: Export Preparedness Index
undertake the monetization of surplus land (EPI) is released by the NITI Aayog. The EPI is a
and building assets of CPSEs and other data-driven effort to identify the core areas
government bodies/organisations. The NLMC crucial for export promotion at the sub-

e
will monetize surplus land and building assets national level (states and union territories).

n
of Central Public Sector Enterprises (CPSEs)

li
and other agencies linked to the Government.
NLMC will act as a repository of best practices Q.28) Ans: B

n
in land monetization, assist and provide Exp: o
technical advice to the Government in the ● Statement 1 is incorrect: It is an autonomous
l.
implementation of the asset monetization body which was set up in 2000 with the
a

program. assistance of the Department of Science and


ri

● Statement 2 is correct: NLMC will also advise Technology.


and support other Government entities ● Statement 2 is correct: Its mission is to help
te

(including CPSEs) in iden tifying their surplus India become a creative and knowledge-based
non-core assets and monetizing them in a society by expanding policy and institutional
a

professional and efficient manner to generate space for grassroots technological innovators.
m

maximum value realization. In these cases It helps grassroot innovators and outstanding
c

(e.g., on-going CPSEs and listed CPSEs under traditional knowledge holders get due
strategic disinvestment), NLMC will undertake recognition, respect and reward for their
s

surplus land asset monetization as an agency innovations.


p

function. It is expected that NLMC will act as a


.u

repository of best practices in land Q.29) Ans: B


monetization, assist and provide technical Exp:
w

advice to Government in implementation of ● Statement 1 is incorrect : It is a subsidiary


w

asset monetization programme. organ of the UN General Assembly and works


as a sexual and reproductive health agency.
w

Q.26) Ans: C The UN Economic and Social Council (ECOSOC)

DPP 2023 DAY 202 21


https://upscmaterial.online/
Download From - https://upscmaterial.online/

Contact us :info@onlyias.com

OnlyIAS Nothing Else Visit :dpp.onlyias.in


Contact : +91-7007 931 912

establishes its mandate. UNFPA works directly Q.31) Ans: C


to tackle Sustainable Development Goal 3 on Exp:
health, Goal 4 on education and Goal 5 on ● Cashless Economy: This type of economic
gender equality. system is based on transactions made through
● Statement 2 is correct: UNFPA is the world's credit card, debit card, digital wallets and
largest multilateral source of funding for modes. Limited amounts of cash transactions
population and reproductive health programs. take place in cashless economies. In this type
The Fund works with governments and non- of economy, the electronic representation of
governmental organizations in over 150 money is encouraged over transactions by
countries with the support of the international coins or physical notes.
community, supporting programs that help The National Payments Corporation of India
women, men and young people: (NPCI) created the Unified Payments Interface
1. voluntarily plan and have the number of (UPI) which is a payment platform in which
children they desire and to avoid unwanted funds can be transferred between the banks
pregnancies effortlessly. It created the digital infrastructure
2. undergo safe pregnancy and childbirth for private players to come and offer apps for
3. avoid spreading sexually transmitted payment services to the citizens. PhonePe,
infections Paytms, mobiquicks are examples. The private
4. decrease violence against women players also helped in popularising the
5. increase the equality of women digitisation in the economy.
6. encouraging the use of birth control The government and the private sector have

e
also started using Big Data techniques for

n
Q.30) Ans: C improving the financial services provided by

li
Exp: them. Credit offerings are now being based on
● Option C is correct: NITI Aayog launched the the transaction history of the consumers.

n
State Energy and Climate Index (SECI). It is the o
first index that aims to track the efforts made ● Merits of Cashless Economy:
l.
by states and UTs in the climate and energy 1. It reduces the tax-burden of the population
a

sector. The parameters of the index have been due to increased overall tax revenue of the
ri

devised keeping in mind India’s goals for government.


climate change and clean energy transition. 2. Transparency in economic transactions
te

● Based on the outcome of SECI scores, states reduces instances of black economy, tax
and union territories have been categorised avoidance, and money laundering.
a

into three groups -- front runners, achievers, 3. Reduction in the cost of printing the cash and
m

and aspirants. handling the cost associated with cash for the
c

1. Top Performers: Gujarat, Kerala and Punjab banks.


have been adjudged as top three performer 4. Digitalization has the potential to transform
s

states in the NITI Aayog’s SECI. conventional banking and enhance the banking
p

2. The top three performers among smaller outreach.


.u

states are Goa, Tripura and Manipur. 5. Macro-economically, it enhances efficiency


3. Unsatisfactory Performance: States like and transparency by reducing Transaction
w

Chhattisgarh, Madhya Pradesh and Jharkhand Costs, a factor in the Corruption Perception
w

were placed at the bottom. Index.


6. It has a potential positive influence on growth
w

and distributive social equity.

DPP 2023 DAY 202 22


https://upscmaterial.online/
Download From - https://upscmaterial.online/

Contact us :info@onlyias.com

OnlyIAS Nothing Else Visit :dpp.onlyias.in


Contact : +91-7007 931 912

7. It makes transactions more convenient rather pillar plan - inclusive framework tax deal on
than carrying the bulk of cash saving the Base Erosion and Profit Shifting (BEPS)- seeks
opportunity cost of time for consumers. to reform international tax rules and ensure
8. The World Bank’s World Development Report- that multinational enterprises pay their fair
2016 envisages that digital technologies have share wherever they operate. The signatories
boosted growth, expanded opportunities, and of the plan amounted to 130 countries and
improved service delivery. jurisdictions, representing more than 90% of
global GDP. The new framework seeks to
● Demerits of Cashless Economy address the tax challenges arising from the
1. India has a low Internet penetration of digitalisation of economies. It also seeks to
34.8%(2016), according to the Internet Live address concerns over cross-border profit
Stats, and only 26.3% of all mobile phone users shifting and bring in subject-to-tax rule to stop
have a smartphone (2015), as per Statista treaty shopping. Treaty shopping is an attempt
figures. Besides the practical difficulty of going by a person to indirectly access the benefits of
digital, “a bigger block is the psychological a tax treaty between two countries without
shift. being a resident of any of those.
Hence it will promote digital inequality. ● Two Pillar Plan: Pillar One: It will ensure a
2. It will increase power consumption as major fairer distribution of profits and taxing rights
components of the Cashless Economy such as among countries with respect to the largest
Mobile Phones, Over the Top devices need MNEs, including digital companies. It would
more power. re-allocate some taxing rights over MNEs from

e
3. Overspending: their home countries to the markets where

n
While there is no denying the convenience of they have business activities and earn profits,

li
card or mobile wallet transactions, it could regardless of whether firms have a physical
open a spending trap for an unsuspecting presence there. According to OECD, more than

n
population. According to behavioural finance USD 100 billion of profit are expected to be
o
theorists, the pain of parting with money is felt reallocated to market jurisdictions each year.
l.
more acutely if you use physical cash instead of Pillar Two: It is about minimum tax and
a

a card. subject-to-tax rules (All sources of income


ri

4. Cyber Security liable to tax without taking account of tax


India will have to tighten its cybersecurity allowances). It seeks to put a minimum
te

owing to the instances of hacking into banks standard tax rate among countries through a
and personal accounts occurring globally. A global minimum corporate tax rate, currently
a

country like India, being at its nascent stage in proposed at 15%. This is expected to generate
m

terms of cybersecurity might be more an additional USD 150 billion in tax revenues.
c

vulnerable to such threats.


Q.33) Ans: C
s

Exp:
p

Q.32) Ans: D ➢ In order to promote the Ease of Doing


.u

Exp: Business, the Ministry of Micro, Small and


● Option D is correct : India and the majority of Medium Enterprises (MSME) has introduced
w

the members of OECD-G20 Inclusive various initiatives including online filing of


w

Framework on Base Erosion and Profit Shifting UdyogAadhaar Memorandum (UAM). The
(BEPS) have joined a new two-pillar plan to Ministry has also taken the following steps:
w

reform international taxation rules. The two-

DPP 2023 DAY 202 23


https://upscmaterial.online/
Download From - https://upscmaterial.online/

Contact us :info@onlyias.com

OnlyIAS Nothing Else Visit :dpp.onlyias.in


Contact : +91-7007 931 912

i. MSME SAMADHAAN Portal- for former Deputy Governor of the Reserve Bank of
empowering micro and small entrepreneurs India (RBI) Usha Thorat.The committee is
across the country to directly register their mandated to advise Sebi on issues related to
cases relating to delayed payments. regulation and development of the mutual fund
ii. MSME SAMBANDH Portal- to help in industry. It can also advise the regulator on
monitoring the implementation of public disclosure requirements and measures required
procurement policy for micro and small for a change in the legal framework to introduce
enterprises. simplification and transparency in mutual fund
iii. MSME SAMPARK Portal – A digital platform regulations.
wherein jobseekers (passed out
trainees/students of MSME Technology Q.35) Ans: A
Centres) and recruiters get connected. Exp:
iv. Digital Payments- to pass on the benefits of ● The Capital markets regulator Securities and
the schemes of the Ministry of MSME Exchange Board of India (SEBI) has introduced
through digital payment gateway. T+1 settlement cycle for completion of share
transactions on optional basis in a move to
enhance market liquidity.SEBI has allowed stock
exchanges to start the T+1 system as an option
Q.34) Ans: C
in place of T+2. T+1 (T+2) are abbreviations that
Exp:
refer to the settlement date of security
● Market regulator Sebi has reconstituted its
transactions. The “T” stands for transaction
advisory committee on leveraging regulatory

e
date, which is the day the transaction takes
and technology solutions (ALeRTS). The seven-

n
place. T+1 means that settlements will have to
member panel will now be headed by Sunil

li
be cleared within one day of the actual
Bajpai.The terms of the committee is to
transactions taking place. Currently, trades on

n
recommend future roadmaps and
the Indian stock exchanges are settled in two
o
improvements in the various on-going
working days after the transaction is done
l.
technology projects. While they are also to guide
(T+2).In April 2002, stock exchanges had
SEBI in designing and framing requirements for
a

introduced a T+3 rolling settlement cycle. This


the various in-house systems.
ri

was shortened to T+2 from April 1, 2003. T+2


● SEBI has constituted an advisory committee for
means if an investor sells shares on Tuesday,
te

advising on ESG (environment, social and


settlement of the trade takes place in two
governance) related matters in the securities
working days(T+2). The broker who handles the
a

market.The committee will be headed by


trade will get the money on Thursday but will
m

Navneet Munot.The terms of reference of the


credit the amount in the investor’s account only
committee include enhancements in business
c

by Friday.
responsibility and sustainability report, ESG
Benefits of T+1 Settlement:
s

ratings and ESG investing.The panel will be


■ A shortened cycle not only reduces
p

responsible for reviewing leadership indicators


settlement time but also reduces and
.u

that may be made essential - including those


frees up the capital required to
related to value chain and developing sector
w

collateralise that risk.


specific sustainability disclosures.
● Markets regulator Sebi has restructured its ■ Secondly, it will provide liquidity to the
w

advisory committee on mutual funds. The 25- investors as they get their funds for the
shares sold/ credited to their account
w

member advisory committee will be chaired by


earlier.
DPP 2023 DAY 202 24
https://upscmaterial.online/
Download From - https://upscmaterial.online/

Contact us :info@onlyias.com

OnlyIAS Nothing Else Visit :dpp.onlyias.in


Contact : +91-7007 931 912

■ Thirdly, it reduces the number of ratio lower than one are unable to meet their
outstanding unsettled trades at any interest obligations from their income. Such
instant, and thus decreases the unsettled firms are categorized as zombies.
exposure to Clearing Corporation by 50%.
■ Lastly, a shortened settlement cycle will
also help in reducing systemic risk. Q.37) Ans: C
Exp:
● Option C is correct : The Gini index, or Gini
coefficient, measures income distribution
Q.36) Ans: B
across a population. Developed by the Italian
Exp:
statistician Corrado Gini in 1912, it often
● Statement 1 is correct : The Interest Coverage
serves as a gauge of economic inequality,
Ratio (ICR) is a financial ratio that is used to
measuring income distribution or, less
determine how well a company can pay the
commonly, wealth distribution among a
interest on its outstanding debts. The ICR is
population. The coefficient ranges from 0 (or
commonly used by lenders, creditors, and
0%) to 1 (or 100%), with 0 representing perfect
investors to determine the riskiness of
equality and 1 representing perfect inequality.
lending capital to a company. The interest
Values over 1 are theoretically possible due to
coverage ratio is also called the “times interest
negative income or wealth. The Gini index is a
earned” ratio. The interest coverage ratio is
measure of the distribution of income across a
calculated by dividing a company's earnings
population. A higher Gini index indicates
before interest and taxes (EBIT) by its interest

e
greater inequality, with high-income
expense during a given period.The interest

n
individuals receiving much larger percentages
coverage ratio is used to measure how well a
of the population's total income. Global

li
firm can pay the interest due on outstanding
inequality, as measured by the Gini index, has

n
debt. The interest coverage ratio is calculated
steadily increased over the past few centuries
o
by dividing a company's earnings before
and spiked during the COVID-19 pandemic.
interest and taxes (EBIT) by its interest expense
l.
Because of data and other limitations, the Gini
during a given period. The "coverage" in the
a

index may overstate income inequality and can


interest coverage ratio stands for the length of
ri

obscure important information about income


time—typically the number of quarters or
distribution. The The Credit Suisse Global
te

fiscal years—for which interest payments can


Wealth Report highlighted that India's Gini
be made with the company's currently
coefficient has steadily increased from 74.7 in
a

available earnings. In simpler terms, it


2000 to 82.3 in 2021. India added 107,000
m

represents how many times the company can


millionaires in 2021 and accounted for at least
pay its obligations using its earnings.
1% of total global millionaires.
c

● Statement 2 is not correct : Companies need


s

to have good profit to cover interest payments.


Q.38) Ans: B
p

A high-Interest Coverage Ratio is advised to


Exp:
.u

meet its interest obligations and in order to


● Statement 1 is correct: Card Tokenization refers
survive future financial hardships (that may
to replacement of actual credit and debit card
w

arise). If a company has a low-interest


details with an alternate code called the
coverage ratio, there’s a high chance that the
w

“token”, which will be unique for a combination


company won’t be able to service its debt. This
of card, token requestor and device.A tokenized
w

will put the firm at risk of insolvency or


card transaction is considered safer as the actual
bankruptcy. Firms with an interest coverage
DPP 2023 DAY 202 25
https://upscmaterial.online/
Download From - https://upscmaterial.online/

Contact us :info@onlyias.com

OnlyIAS Nothing Else Visit :dpp.onlyias.in


Contact : +91-7007 931 912

card details are not shared with the merchant India (NPCI), which oversees the digital
during transaction processing. Customers who payments ecosystem in India, has launched e-
do not have the tokenisation facility will have to RUPI, a voucher-based payments system to
key in their name, 16-digit card number, expiry promote cashless transactions. It has been
date and CVV each time they order something developed in collaboration with the Department
online. This could be cumbersome exercise.This of Financial Services, Ministry of Health & Family
essentially means that a customer's card Welfare and National Health Authority.
information will no longer be available on any ● Statement 2 is correct: e-RUPI has a QR code or
Merchant, Payment Gateway, or 3rd party that SMS string-based e-Voucher, which is delivered
helps in the processing of digital transactions to the mobile of the beneficiaries. e-RUPI does
today. not require the beneficiary to have a bank
● Statement 2 is incorrect: Tokenization is account, a major distinguishing feature as
applicable only for Domestic transactions. compared to other digital payment forms. It
Tokenization of the card is absolutely free and ensures an easy, contactless two-step
can be availed by anyone.Three steps have to be redemption process that does not require
completed for smooth implementation of sharing of personal details either. e-RUPI is
tokenization: operable on basic phones also, and hence it can
be used by persons who do not own smart-
➢ Token provisioning: the consumer’s card
phones or in places that lack internet
number should be convertible into a token,
connection. The e-RUPI prepaid digital voucher,
which means the card networks have to be
developed by the National Payments
ready with the relevant infrastructure.

e
Corporation of India (NPCI)

n
➢ Token processing: Consumers should be able ● Statement 3 is incorrect: e-RUPI is expected to

li
to complete their transaction successfully play a major role in strengthening Direct-Benefit
Transfer and making it more transparent. Since,

n
through the tokens.
there is no need for physical issuance of
o
➢ Scale-up for multiple use cases: Consumers vouchers, it will also lead to some cost savings as
l.
should be able to use the token for things like well. It is a person-specific and purpose-specific
a

refunds, EMIs, recurring payments, offers, cashless voucher and can be used by
ri

promotions, guest checkouts etc. individuals, corporates or governments. It will


connect sponsors of the services with
te

beneficiaries and service providers digitally


Q.39) Ans: A
without any physical interface.
a

Exp:
m

● Statement 1 is correct: Recently,the RBI has


hiked the cap on e-Rupi prepaid digital vouchers
c

Q.40) Ans: C
from Rs 10,000 to Rs 1 lakh and enabled the
Exp:
s

instrument for multiple transactions.e-RUPI is a


● The decision to allow international trade in
p

cashless and contactless digital payments


rupees is aimed at easing trade with Sri Lanka,
.u

medium, which will be delivered to mobile


which is running low on forex reserves, and
phones of beneficiaries in the form of an SMS
Russia, which cannot make payments in US
w

string or a QR code. It is a person and purpose-


dollars due to sanctions by the West.To accept
specific digital payment solution which is a
w

payments in rupees, authorized dealer banks


cashless and contactless instrument for digital
will be able to open special Rupee Vostro
w

payment.The National Payments Corporation of


accounts.A Rupee Vostro account is a foreign

DPP 2023 DAY 202 26


https://upscmaterial.online/
Download From - https://upscmaterial.online/

Contact us :info@onlyias.com

OnlyIAS Nothing Else Visit :dpp.onlyias.in


Contact : +91-7007 931 912

bank’s account with an Indian bank in rupees requirements.The six-member committee will be
in India.This move will help reduce India’s headed by Sudarshan Sen, former executive
dependency on US dollars. The recent director, RBI.The panel will review the existing
Ukraine-Russia crisis and sanctions on Russia legal and regulatory framework applicable to ARCs
was a real eye opener for most countries who and recommend measures to improve efficacy of
are now trying to lower their dependency on ARCs. It will also review the role of ARCs in the
the US dollar. Moreover, since India runs a resolution of stressed assets, including under the
trade deficit – its imports are greater than Insolvency and Bankruptcy Code (IBC) and give
exports – settling trades in rupees will also suggestions for improving liquidity in and trading
save dollar outflows. At a time when the of security receipts. Besides, it has also been asked
rupee’s value is declining every week against to review the business models of ARCs.
the US dollar, saving dollar outflows becomes
even more critical for the RBI. Several Q.42) Ans: D
countries including Sri Lanka and some in Exp:
Africa and Latin America are facing forex ● Barbell Strategy combines a bouquet of safety-
shortage. As such, the new mechanism will nets to cushion the impact on vulnerable
help India promote its exports.The move sections of society/business, with a flexible
would also reduce the risk of forex fluctuation policy response based on a Bayesian updating of
specially looking at the Euro-rupee parity. It information. As explained in last year’s
will also help stabilize the rupee.It will also Economic Survey, this is a common strategy used
help buy discounted crude oil from Russia, in financial markets to deal with extreme

e
which now accounts for 10% of all imported uncertainty by combining two seemingly

n
crude. RBI’s decision may not benefit the disparate legs. As some readers will have

li
external account immediately, but over the guessed, the iterative leg of this strategy is the
medium term, demand for dollars may come same as the “Agile'' approach that uses

n
down and this move will help narrow the trade feedback-loops, and real-time adjustment. The
o
deficit. It will slow down the depreciation of barbell strategy is an approach to uncertainty
l.
the rupee as It will promote the growth of (risk) that uses two extremes – like weights on
a

global trade with emphasis on exports from the opposite ends of a barbell – to avoid ruin and
ri

India and to support the increasing interest of simultaneously expose yourself to a speculative
the global trading community in INR. Amid upside. On one end of the barbell is extreme risk
te

ongoing rupee weakness, this mechanism aims aversion (safety). On the other end is extreme
at reducing demand for foreign exchange, by risk loving (speculation). What you avoid is the
a

promoting rupee settlement of trade flows. “middle” of the barbell – a moderate risk
m

Since India imports more than it exports. It will attitude that is highly prone to error. It is a
c

enhance forex inflows and as a step to method that consists of taking both a defensive
stabilize the rupee. attitude and an excessively aggressive one at the
s

same time, by protecting assets from all sources


p

Q.41) Ans: C of uncertainty while allocating a small portion


.u

Exp: for high-risk strategies.(a financial market


● The RBI set up a committee to undertake a concept). Example-A retail investor secures 80
w

comprehensive review of the working of Asset percent of his capital by investing in FDs and
w

Reconstruction Companies (ARCs) in the financial invests the remaining in speculative stocks or
sector ecosystem and recommend suitable crypto-currencies. If Bitcoin goes to zero,
w

measures for enabling them to meet the growing investors lose just 20 percent of the money. But

DPP 2023 DAY 202 27


https://upscmaterial.online/
Download From - https://upscmaterial.online/

Contact us :info@onlyias.com

OnlyIAS Nothing Else Visit :dpp.onlyias.in


Contact : +91-7007 931 912

a few lucky ones saw the Bitcoin returns up by a bulk of the guar in split or flour form called guar
hundred times. The idea is to focus on extreme gum that is mainly used by the oil and gas
ends and avoid medium-risk strategies. industry in shale oil facilities. Guar gum is also
consumed by food, pharma, paint and other
industrial sectors. India’s guar gum exports have
Q.43) Ans: C fallen 30 per cent in the last two years.
Exp: ● Statement 2 is incorrect: National Commodity
Retail inflation based on the Consumer Price and Derivatives Exchange has launched futures
Index (CPI).Consumer Price Index/Retail contract on NCDEX Guarex, India’s first sectoral
Inflation-has several sub-groups: Food and index in agriculture commodities basket.The
beverages; Pan, tobacco & intoxicants; Clothing first sectoral agriculture index futures contract
& footwear; Fuel and light; Housing and will garner interest from all sets of participants,
Miscellaneous. Conventionally, core inflation is including the physical market participants.
calculated by excluding ‘food and beverages’ NCDEX, incorporated as a public limited
and ‘fuel and light’ groups from overall inflation company on April 23, 2003, is a leading
(CPI-C).In the CPI-C these fuel items excluded agricultural commodity exchange in India.
from core are included in ‘transport and ● Statement 3 is incorrect: India is the world’s
communication’, a subgroup under the largest guar producer accounting for 80-85 per
miscellaneous group. Therefore, conventional cent of the output, while Rajasthan is the
ways of calculating retail core inflation, instead country’s top producer with 80 per cent market
of excluding the volatile fuel items from core share. The index will soon be the benchmark for

e
inflation, continue to include volatile fuel items the entire guar complex, both in the domestic

n
in core inflation. As a result, the fuel price rise and international markets, making it a value

li
continues to impact core inflation. A ‘refined' proposition for eligible foreign entities, to hedge
core inflation was constructed to address this their price risk on Indian bourses.

n
anomaly by excluding main fuel items viz., o
‘petrol for vehicle’, ‘diesel for vehicle’ and
l.
‘lubricants and other fuels for vehicles’, in Q.45) Ans: B
a

addition to ‘food and beverages’ and ‘fuel and Exp:


ri

light’ from the headline retail inflation. Since ● The monetary policy is a collection of financial
June 2020, refined core inflation has been much tools and measures available with the RBI to
te

below the conventional core inflation, safeguard and promote economic growth.
indicating the impact of inflation in fuel items in Monetary policies basically control the overall
a

the conventional core inflation measure. supply of money available to commercial banks
m

and, indirectly, to individual users and


c

companies.Accommodative monetary policy,


Q.44) Ans: B also known as loose credit or easy monetary
s

Exp: policy, occurs when a central bank (such as


p

● Statement 1 is correct: Guarex is an index RBI) attempts to expand the overall money
.u

tracking the price movement in the futures supply to boost the economy when growth is
contracts of its underlying commodities such as slowing.The policy is implemented to allow the
w

guarseed and guar gum refined splits on a real- money supply to rise in line with national
w

time basis. The weightage of guarseed and guar income and the demand for money.These
gum refined splits in the index will be 63 per cent measures are meant to make money less
w

and 37 per cent, respectively. India exports the expensive to borrow and encourage more

DPP 2023 DAY 202 28


https://upscmaterial.online/
Download From - https://upscmaterial.online/

Contact us :info@onlyias.com

OnlyIAS Nothing Else Visit :dpp.onlyias.in


Contact : +91-7007 931 912

spending from consumers and businesses by the RBI. Thus, the reverse repo rate is the
making money less expensive to borrow opposite of the repo rate.
through the lowering of short-term interest
rates. When money is easily accessible through
banks, the money supply in the economy
increases. This leads to increased spending. Q.47) Ans: A
When businesses can easily borrow money, Exp:
they have more funds to expand operations ● Option A is correct: REvil (Ransomware Evil;
and hire more workers, which means that the also known as Sodinokibi) was a Russia-based
unemployment rate will decrease.On the or Russian-speaking private ransomware-as-a-
other hand, people and businesses tend to service (RaaS) operation. REvil’s name is an
save less when the economy is stimulated due amalgam of “ransomware” and “evil”. It is a
to the low savings interest rates offered by Russia-based hacking Organization. Group
banks. REvil would steal data from computers, lock
the victims out of their computers, and then
threaten to release stolen data by auctioning it
Q.46) Ans: A
off. Ransomware group REvil has been
Exp:
dismantled by the Russian authorities at the
● Statement 1 is incorrect: Liquidity Adjustment
request of US government agencies.
Facility(LaF) Corridor indicates the difference
between the Repo and Reverse Repo with
Q.48) Ans: C
reverse repo rate as the lower bound of the

e
Exp:
liquidity adjustment facility (LAF) corridor and

n
● On 8 August 1967, the Foreign Ministers of
Repo as the upper bound. RBI will now use

li
Indonesia, Malaysia, the Philippines, Singapore
standing deposit facility (SDF) as the floor rate
signed a document to establish the Association

n
for Liquidity Adjustment Facility (LAF) corridor,
of Southeast Asian Nations (ASEAN).
o
instead of the reverse repo and through it seeks
● Statement 1 is incorrect: It was established as
l.
to suck out excess liquidity in the system, but
per the rationales and objectives under
without offering any collateral to banks.
a

Bangkok Declaration. The ASEAN Summit is the


● Statement 2 is correct: A liquidity adjustment
ri

highest policy-making body in ASEAN


facility (LAF) is a tool used in monetary policy,
comprising the Head of States or Government
te

which enables banks to borrow money through


of ASEAN Member States. The ASEAN Summit
repurchase agreements or banks to lend to the
is held twice annually at a time to be
a

RBI using reverse repo contracts.The


determined by the Chair of the ASEAN Summit
m

Repurchase agreement (Repo) and the Reverse


in consultation with the other ASEAN Member
repo agreement are two key tools used by the
c

States.
Reserve Bank of India (RBI) to control the money
● Statement 2 is correct: ASEAN Defence
s

supply.The repo rate is the interest rate at which


Ministers Meeting (ADMM)-Plus Meeting: The
p

a country's central bank(in India, the RBI) loans


ADMM-Plus is a platform for ASEAN and its 8
.u

money to the commercial banks in the event of


Dialogue Partners (Australia, China, India,
a shortfall of funds. The central bank purchases
Japan, New Zealand, ROK, Russian Federation,
w

the security in this case.The reverse repo rate is


and the United States) to strengthen security
the interest rate paid by the RBI to commercial
w

and defence cooperation for peace, stability,


banks that park excess “liquidity”(money) with
and development in the region.
w

DPP 2023 DAY 202 29


https://upscmaterial.online/
Download From - https://upscmaterial.online/

Contact us :info@onlyias.com

OnlyIAS Nothing Else Visit :dpp.onlyias.in


Contact : +91-7007 931 912

● Statement 3 is correct: The year 2022 marks 30 interaction between the SCO member states in
years of ASEAN-India relations and it has been the fight against terrorism, extremism and
designated as ASEAN-India Friendship Year by separatism.
the leaders in October 2021. ● The main functions of SCO-RATS are
● Statement 4 is incorrect: ASEAN is India’s coordination and information sharing.
fourth largest trading partner. India is ASEAN's ● As a member, India has actively participated in
seventh. Over 20% of India's outbound the activities of SCO-RATS.
investments go to ASEAN. Led by Singapore, ● India’s permanent membership would enable
ASEAN is India's leading source of investments. it to generate greater understanding among
members for its perspective.
Q.49) Ans: C
Exp: Q.50) Ans: C
● Statement 1 is correct: SCO is a permanent Exp:
intergovernmental international organization. ● Recently, a sharp and sudden spike in fuel
It’s a Eurasian political, economic and military prices triggered a national crisis in Kazakhstan
organization aiming to maintain peace, ● Statement 1 is incorrect: Sandwiched
security and stability in the region. between Russia and China, Kazakhstan is the
● Statement 2 is incorrect: SCO was created in world’s largest landlocked country, bigger than
2001 but India is not a founding member of the whole of Western Europe.
SCO. India and Pakistan became members in ● Statement 2 is correct: It has vast mineral
2017. Iran has signed a Memorandum of resources, with 3% of global oil reserves and

e
Obligations to become a permanent member important coal and gas sectors. It is the top

n
of the Shanghai Cooperation Organisation global producer of uranium, which jumped in

li
(SCO). price by 8% after the unrest. The country is also
● Statement 3 is correct: The city of Varanasi the world’s second-largest miner of bitcoin.

n
has been nominated as the first-ever SCO ● Statement 3 is correct: Recently there was
o
Tourism and Cultural Capital during the period unrest in Kazakhstan due to rising fuel prices.
l.
2022-2023 at the 22nd Meeting of Shanghai Angry Kazakhs first took to the streets after
a

Cooperation Organization (SCO) Council of fuel prices doubled in the oil-rich Central Asian
ri

Heads of State in Samarkand, Uzbekistan on nation when the government lifted price caps
September 16, 2022. Prime Minister Shri for Liquified Petroleum Gas (LPG), commonly
te

Narendra Modi had participated in the used in vehicles. Protesters demanded the
Summit. The nomination of Varanasi as the resignation of the government and lowering of
a

first ever SCO Tourism and Cultural Capital will LPG prices
m

promote tourism, cultural and humanitarian


c

exchanges between India and the SCO member Q.51) Ans: C


Countries. It also underlines India’s ancient Exp:
s

civilizational links with Member States of SCO, ● Option C is correct: Red Corner Notice - It is an
p

especially the Central Asian Republics. international document against wanted


.u

persons, requesting law enforcement agencies


Extra Edge by OnlyIAS around the world to locate and temporarily
w

Regional Anti-Terrorist Structure of Shanghai detain a suspect until their extradition,


w

Cooperation Organization (RATS-SCO) surrender, or other legal action. It is not an


● SCO-RATS is a permanent body of the SCO and international arrest warrant.
w

is intended to facilitate coordination and What is Interpol?

DPP 2023 DAY 202 30


https://upscmaterial.online/
Download From - https://upscmaterial.online/

Contact us :info@onlyias.com

OnlyIAS Nothing Else Visit :dpp.onlyias.in


Contact : +91-7007 931 912

Interpol, or International Criminal Police UN charter, each member is legally bound to


Organisation, is an inter-governmental body pay their individual share for peacekeeping.
with 194 member countries. It is The expenses for a peacekeeping operation is
headquartered in Lyon, France, and has offices divided by the United Nations General
in several countries around the world. Assembly based on a formula that takes into
Founded in 1923, it facilitates global police account the economic condition of member
cooperation to fight international crime. states as one of the factors.
What does a Red Corner Notice contain? ● Statement 2 is not correct : The UN
There are primarily two types of information in Peacekeeping force is supplemented by
a Red Corner Notice: personnel from member nations. They are
1. Information to identify the wanted added to the force on a volunteer basis. As of
person, such as their name, date of late about, 100,000 personnel serve in the UN
birth, nationality, photographs, etc. Peacekeeping forces. Among the largest
2. Information about the crime that they individual contributors, India, Pakistan and
are wanted for. These crimes typically Bangladesh top the list with 8000 units each.
include murder, rape, child abuse or As the peacekeeping forces are being created,
armed robbery, according to the some behind-the-curtain-diplomatic actions
Interpol’s website. are taken by the UN. The size and strength of
Indians and Red Corner notices: Brar, a the force are decided upon by the government
resident of Sri Muktsar Sahib who had gone to in whose territory the peacekeeping force will
Canada in 2017, is one of the latest Indian- be deployed to. Along with this the rules of

e
origin persons to face a Red Corner Notice. In engagement are formulated and agreed upon

n
the past, such a notice has been issued against by the parties involved with approval from the

li
fugitive diamantaire Nirav Modi and his wife UNSC.
Ami Modi, fugitive businessman Mehul Choksi,

n
gangster Ravi Pujari, and former D-Company Q.53) Ans: B o
gang member Chhota Rajan. Exp:
l.
● Statement 1 is not correct : ASEAN was
a

established on 8th August 1967 in Bangkok,


ri

Thailand with the signing of the Bangkok


Q.52) Ans: B Declaration (a.k.a ASEAN Declaration) by the
te

Exp: founding fathers of the countries of Indonesia,


● Statement 1 is correct: The United Nations Malaysia, Thailand, Singapore, and the
a

Peacekeeping operations are policing and Philippines. The preceding organisation was
m

peacebuilding actions carried out by the UN to the Association of Southeast Asia (ASA)
bring order and stability in war torn nations.
c

comprising Thailand, the Philippines, and


The UN Peacekeepers are known as ‘Blue Malaysia. Five other nations joined the ASEAN
s

Helmets’ or ‘Blue Berets’. Its personnel in subsequent years making the current
p

consist of soldiers and military officers, police membership to ten countries.


.u

officers and civilian personnel from many ● Statement 2 is not correct : ASEAN Plus Three:
countries. A peacekeeping mission is funded The consultative group initiated in 1997
w

collectively by the United Nations member brings together ASEAN’s ten members, China,
w

states, while the establishment and Japan, and South Korea. East Asia Summit
maintenance of its operations are decided by (EAS): First held in 2005, the summit seeks to
w

the United Nations Security Council. As per the promote security and prosperity in the region

DPP 2023 DAY 202 31


https://upscmaterial.online/
Download From - https://upscmaterial.online/

Contact us :info@onlyias.com

OnlyIAS Nothing Else Visit :dpp.onlyias.in


Contact : +91-7007 931 912

and is usually attended by the heads of state a multilateral development bank operated by
from ASEAN, Australia, China, India, Japan, the BRICS states (Brazil, Russia, India, China
New Zealand, Russia, South Korea, and the and South Africa). It was agreed to by BRICS
United States. ASEAN plays a central role as the leaders at the 5th BRICS summit held in
agenda-setter.ASEAN Plus Six: The group Durban, South Africa in 2013. It was
includes ASEAN Plus Three as well as India, established in 2014, at the 6th BRICS Summit
Australia, and New Zealand. at Fortaleza, Brazil. The bank is set up to foster
● Statement 3 is correct : ASEAN-India greater financial and development
Friendship Year : The year 2022 also marks the cooperation among the five emerging markets.
30th Anniversary of ASEAN-India Dialogue ● Statement 2 is not correct : In 2018, the NDB
Relations & the 10th anniversary of India’s received observer status in the United Nations
Strategic Partnership with ASEAN. So, the General Assembly, establishing a firm basis for
year has been designated as the ASEAN-India active and fruitful cooperation with the UN.
Friendship Year. The Special ASEAN-India Unlike the World Bank, which assigns votes
Foreign Ministers’ Meeting (SAIFMM) was based on capital share, in the New
hosted by India in New Delhi recently. This is Development Bank each participant country
important as it will strengthen the ASEAN-India will be assigned one vote, and none of the
Strategic Partnership. countries will have veto power. The initial
● Plan of Action 2021-2025: The ongoing India- subscribed capital of the bank was equally
ASEAN collaboration is guided by the Plan of distributed among the founding members
Action 2021-2025 which was adopted in 2020. (Brazil, Russia, India, People's Republic of

e
The plan of Action envisages greater China, South Africa). The Agreement on the

n
cooperation in areas ranging from trade to NDB specifies that every member will have one

li
maritime security and counter-terrorism. vote no one would have any veto powers . The
Bank will mobilise resources for infrastructure

n
Q.54) Ans: D and sustainable development projects in BRICS
o
Exp: and other emerging economies and developing
l.
● Statement 1 is not correct : The New countries, to supplement existing efforts of
a

Development Bank (NDB), formerly referred multilateral and regional financial institutions
ri

to as the BRICS Development Bank, is a for global growth and development.


multilateral development bank established by
te

the BRICS states (Brazil, Russia, India, China Q.55) Ans: B


and South Africa). According to the Agreement Exp:
a

on the NDB, "the Bank shall support public or ● Statement 1 is not correct : The Organization
m

private projects through loans, guarantees, of the Petroleum Exporting Countries (OPEC)
c

equity participation and other financial is a permanent, intergovernmental


instruments." Moreover, the NDB "shall organization, created at the Baghdad
s

cooperate with international organizations and Conference in 1960, by Iran, Iraq, Kuwait,
p

other financial entities, and provide technical Saudi Arabia, and Venezuela. It aims to
.u

assistance for projects to be supported by the manage the supply of oil in an effort to set the
Bank.The bank is headquartered in Shanghai, price of oil in the world market, in order to
w

China. The first regional office of the NDB is in avoid fluctuations that might affect the
w

Johannesburg, South Africa. The second economies of both producing and purchasing
regional office was established in 2019 in São countries. It is headquartered in Vienna,
w

Paulo, Brazil, followed by Moscow, Russia.It is Austria.

DPP 2023 DAY 202 32


https://upscmaterial.online/
Download From - https://upscmaterial.online/

Contact us :info@onlyias.com

OnlyIAS Nothing Else Visit :dpp.onlyias.in


Contact : +91-7007 931 912

● Statement 2 is correct : OPEC membership is Upper Credit Tranche (UCT) IMF-supported


open to any country that is a substantial program. The Common Framework represents
exporter of oil and which shares the ideals of a step-change for official creditors, bringing
the organization.Gabon terminated its together the Paris Club and G20 official
membership in January 1995. However, it bilateral creditors in a coordinated process.
rejoined the Organization in July 2016. Current Such an approach will allow addressing
OPEC members are Algeria, Angola, Equatorial solvency challenges with a long-term
Guinea, Gabon, Iran, Iraq, Kuwait, Libya, perspective, ensuring the participation of
Nigeria, the Republic of the Congo, Saudi private-sector creditors and of other official
Arabia, the United Arab Emirates and creditors through the comparability of the
Venezuela. Meanwhile, Ecuador, Indonesia treatment clause included in the multilateral
and Qatar are former OPEC members. agreement, which implies that they provide
● Statement 3 is not correct: The non-OPEC debt treatments on terms as least as
countries which export crude oil along with favourable.
the 14 OPECs are termed as OPEC plus
countries. OPEC plus countries include Q.57) Ans: C
Azerbaijan, Bahrain, Brunei, Kazakhstan, Exp:
Malaysia, Mexico, Oman, Russia, South Sudan, ● Statement 1 is not correct : The G20 was
and Sudan. Saudi and Russia, both have been formed in 1999 in the backdrop of the
at the heart of a three-year alliance of oil financial crisis of the late 1990s that hit East
producers known as OPEC Plus — which now Asia and Southeast Asia in particular. It aims

e
includes 11 OPEC members and 10 non-OPEC to secure global financial stability by involving

n
nations — that aims to shore up oil prices with middle-income countries. Together, the G20

li
production cuts. countries include 60 % of the world’s
population, 80 % of global GDP, and 75 % of

n
global trade. o
Q.56) Ans: A ● Members: Argentina, Australia, Brazil, Canada,
l.
Exp: China, France, Germany, India, Indonesia, Italy,
a

● Option A is correct : The Common Framework Japan, Republic of Korea, Mexico, Russia, Saudi
ri

for debt treatment beyond the DSSI (Common Arabia, South Africa, Turkey, the United
Framework) is an initiative of the G20, Kingdom, the United States and the EU. Spain
te

together with the Paris Club to support Low- is invited as a permanent guest.
Income Countries with unsustainable debt in ● Statement 2 is not correct : The G20 has no
a

a structural manner. The Common Framework permanent secretariat. The agenda and work
m

considers debt treatment, on a case-by-case are coordinated by representatives of the G20


c

basis, driven by requests from eligible debtor countries, known as ‘Sherpas’, who work
countries. In response to a request for debt together with the finance ministers and
s

treatment, a Creditor Committee is convened. governors of the central banks. The primary
p

Negotiations are supported by the IMF and the mandate of the grouping is for International
.u

World Bank, including through their Debt Economic cooperation with particular
Sustainability Analysis. The idea is that the emphasis to prevent future financial crises
w

debt treatment under the Common across the world. It plays a significant role in
w

Framework should be accompanied by reforms shaping the global economic agenda. From
ensuring the future sustainability of public 1999-2008 the forum exalted from a grouping
w

debt, and consistent with the parameters of an

DPP 2023 DAY 202 33


https://upscmaterial.online/
Download From - https://upscmaterial.online/

Contact us :info@onlyias.com

OnlyIAS Nothing Else Visit :dpp.onlyias.in


Contact : +91-7007 931 912

of Central bank governors and finance platform that facilitates criminal investigation
ministers to Heads of states. of police forces across the globe through
collection and dissemination of information
Q.58) Ans: C received from various police forces. It is
Exp: headquartered in Lyon, France. It keeps track
● Statement 1 is correct: Asian Infrastructure of the movements of criminals and those
Investment Bank (AIIB) is a multilateral under the police radar in various regions and
development bank with a mission to improve tips off police forces which had either sought
social and economic outcomes in Asia and the Interpol’s assistance or which in its opinion
beyond. The Parties (57 founding members) to will benefit from the particulars available with
agreement comprise the Membership of the it. It aims to promote the widest-possible
Bank. It is headquartered in Beijing. The bank mutual assistance between criminal police
started operation after the agreement entered forces.
into force on 25 December 2015, after ● Statement 2 is not correct: All contact of a
ratifications were received from 10 member country’s law enforcement agency with
states holding a total number of 50% of the Interpol is through the highest investigating
initial subscriptions of the Authorized Capital body of the land. The Central Bureau of
Stock. Investigation (CBI) assumes this role in India
● Statement 2 is not correct : China is the largest with one of its senior officers heading its
shareholder with 26.61 % voting shares in the exclusive Wing (the National Central Bureaus)
bank followed by India (7.6%), Russia (6.01%) for collation of information and liaison with the

e
and Germany (4.2 %). The regional members world body.

n
hold 75% of the total voting power in the Bank. ● Statement 3 is not correct: The General

li
● Statement 3 is not correct: The Asian Assembly of the International Criminal Police
Infrastructure Investment Bank (AIIB) is a Organisation (Interpol) is meeting in Delhi for

n
multilateral development bank that aims to four days from October 18, 2022. This is the
o
improve economic and social outcomes in Asia. second time since 1997 the 195 member-
l.
The bank currently has 105 members, strong body is holding such a large conference
a

including 14 prospective members from in India.


ri

around the world. The breakdown of the 105


members by continents are as follows: 42 in
te

Asia, 26 in Europe, 20 in Africa, 8 in Oceania,


8 in South America, and 1 in North America. Q.60) Ans: B
a

The bank started operation after the Exp:


m

agreement entered into force on 25 December ● Option 1 is not correct : The Association of
c

2015, after ratifications were received from 10 Southeast Asian Nations (ASEAN), was
member states holding a total number of 50% established on 8 August 1967 in Bangkok,
s

of the initial subscriptions of the Authorized Thailand, with the signing of the ASEAN
p

Capital Stock Declaration (Bangkok Declaration) by the


.u

Founding Fathers of ASEAN, namely Indonesia,


Malaysia, Philippines, Singapore, and Thailand.
w

Q.59) Ans: A Brunei Darussalam then joined on 7 January


w

Exp: 1984, VietNam on 28 July 1995, Lao PDR and


● Statement 1 is correct: Interpol : It was set up Myanmar on 23 July 1997, and Cambodia on
w

in 1923, as a secure information-sharing 30 April 1999, making up what is today the ten

DPP 2023 DAY 202 34


https://upscmaterial.online/
Download From - https://upscmaterial.online/

Contact us :info@onlyias.com

OnlyIAS Nothing Else Visit :dpp.onlyias.in


Contact : +91-7007 931 912

Member States of ASEAN. India is not a Exp:


member of ASEAN. ● Statement 1 is not correct : Missile
● Option 2 is correct : BIMSTEC : The Bay of Technology Control Regime is an informal and
Bengal Initiative for Multi-Sectoral Technical voluntary partnership, not a binding
and Economic Cooperation (BIMSTEC) is a agreement among 35 countries to prevent the
regional multilateral organisation. Its members proliferation of missile and unmanned aerial
lie in the littoral and adjacent areas of the Bay vehicle technology capable of carrying greater
of Bengal constituting a contiguous regional than 500 kg payload for more than 300 km.
unity. Out of the 7 members, Five are from MECR are voluntary and non-binding
South Asia – Bangladesh; Bhutan ; India ; Nepal agreements created by the major supplier
; Sri Lanka : Two are from Southeast Asia – countries that have agreed to co-operate in
Myanmar; Thailand. BIMSTEC not only their effort to prevent and regulate the
connects South and Southeast Asia, but also transfer of certain military and dual use
the ecologies of the Great Himalayas and the technology.
Bay of Bengal. ● Statement 2 is not correct : The members are
● Option 3 is not correct : The South Asian thus prohibited from supplying such missiles
Association for Regional Cooperation (SAARC) and UAV systems that are controlled by the
was established with the signing of the SAARC MTCR to non-members. It was established in
Charter in Dhaka on 8 December 1985.The idea April 1987 by G-7 countries – USA, UK, France,
of regional cooperation in South Asia was first Germany, Canada, Italy, and Japan. In 1992,
raised in November 1980. After consultations, the focus of the regime extended to on the

e
the foreign secretaries of the seven founding proliferation of missiles for the delivery of all

n
countries—Bangladesh, Bhutan, India, types of weapons of mass destruction (WMD),

li
Maldives, Nepal, Pakistan, and Sri Lanka—met i.e., nuclear, chemical and biological weapons.
for the first time in Colombo in April 1981. It is not a legally-binding treaty.

n
Afghanistan became the newest member of ● Statement 3 is correct : The efforts of non-
o
SAARC at the 13th annual summit in 2005. The proliferation of ballistic missile systems had
l.
Headquarters and Secretariat of the further been strengthened by “The
a

Association are at Kathmandu, Nepal. International Code of Conduct against Ballistic


ri

Myanmar is not a member of SAARC. Missile Proliferation”, also known as the Hague
● Option 4 is correct : East Asia Summit : Code of Conduct (HCOC), which was
te

Established in 2005, it is a forum of 18 regional established on 25 November 2002 as an


leaders for strategic dialogue and cooperation arrangement to prevent the proliferation of
a

on the key political, security, and economic ballistic missiles with 136 UN member
m

challenges facing the Indo-Pacific region. It countries including India. India was inducted
c

comprises the ten member states of the ASEAN into the Missile Technology Control Regime in
(Association of Southeast Asian Nations) which 2016 as the 35th member.
s

are Brunei, Cambodia, Indonesia, Laos,


p

Malaysia, Myanmar, the Philippines,


.u

Singapore, Thailand, and Vietnam, along with 8 Q.62) Ans: B


other countries namely Australia, China, Japan, Exp:
w

India, New Zealand, the Republic of Korea, ● Statement 1 is not correct : International
w

Russia and the USA. Telecommunication Union (ITU) It is an


agency of the United Nations (UN) whose
w

Q.61) Ans: A purpose is to coordinate telecommunication

DPP 2023 DAY 202 35


https://upscmaterial.online/
Download From - https://upscmaterial.online/

Contact us :info@onlyias.com

OnlyIAS Nothing Else Visit :dpp.onlyias.in


Contact : +91-7007 931 912

operations and services throughout the war). The first Geneva Convention protects
world. Originally founded in 1865, as the wounded and sick soldiers on land during war.
International Telegraph Union, the ITU is the The second Geneva Convention protects
oldest existing international organisation. wounded, sick and shipwrecked military
Headquarters are in Geneva, Switzerland. The personnel at sea during war. The third Geneva
ITU consists of three sectors: Convention applies to prisoners of war. The
Radiocommunication (ITU-R) — ensures fourth Geneva Convention affords protection
optimal, fair and rational use of the radio to civilians, including in occupied territory.
frequency (RF) spectrum. Telecommunication Article 3, common to the four Geneva
Standardization (ITU-T) — formulates Conventions, covers situations of non-
recommendations for standardising international armed conflicts. They include
telecommunication operations worldwide. traditional civil wars, internal armed conflicts
Telecommunication Development (ITU-D) — that spill over into other States or internal
assists countries in developing and maintaining conflicts in which a third State or a
internal communication operations. multinational force intervenes alongside the
● Statement 2 is correct: India has been an government. Two Protocols of 1977:
active member of the ITU since 1869 and has Additional to the four 1949 Geneva
been a regular member of the ITU Council Conventions were adopted in 1977. They
since 1952. In November 2018, India was strengthen the protection of victims of
elected as a Member of the ITU Council for international (Protocol I) and non-
another 4-year term (2019-2022). international (Protocol II) armed conflicts and

e
● Statement 3 is correct: There are 193 Member place limits on the way wars are fought. In

n
States of the ITU, including all UN member 2005, a third Additional Protocol was adopted

li
states except the Republic of Palau, plus the creating an additional emblem, the Red
Vatican City. Membership of ITU is open to Crystal, which has the same international

n
only UN members, which may join the Union status as the Red Cross and Red Crescent
o
as Member States, as well as to private emblems.
l.
organisations like carriers, equipment
a

manufacturers, funding bodies, research and Q.64) Ans: D


ri

development organisations and international Exp:


and regional telecommunication ● Option D is correct: The Minsk agreements
te

organisations , which may join ITU as non- were a series of international agreements
voting Sector Members. which sought to end the Donbas war fought
a

between armed Russian separatist groups


m

and Armed Forces of Ukraine, with Russian


c

Q.63) Ans: C regular forces playing a central part. The first,


Exp: known as the Minsk Protocol, was drafted in
s

● Option C is correct : The Geneva Conventions 2014 by the Trilateral Contact Group on
p

(1949) and their Additional Protocols are Ukraine, consisting of Ukraine, Russia, and
.u

international treaties that contain the most the Organization for Security and Co-
important rules limiting the barbarity of war. operation in Europe (OSCE), with mediation by
w

They protect people who do not take part in the leaders of France and Germany in the so-
w

the fighting (civilians, medics, aid workers) called Normandy Format. After extensive talks
and those who can no longer fight (wounded, in Minsk, Belarus, the agreement was signed
w

sick and shipwrecked troops, prisoners of on 5 September 2014 by representatives of the

DPP 2023 DAY 202 36


https://upscmaterial.online/
Download From - https://upscmaterial.online/

Contact us :info@onlyias.com

OnlyIAS Nothing Else Visit :dpp.onlyias.in


Contact : +91-7007 931 912

Trilateral Contact Group and, without ● Statement 2 is not correct : Universal Periodic
recognition of their status, by the then-leaders Review: UPR serves to assess the human rights
of the self-proclaimed Donetsk People's situations in all United Nations Member States.
Republic (DPR) and Luhansk People's Republic Advisory Committee: It serves as the Council’s
(LPR). This agreement followed multiple “think tank” providing it with expertise and
previous attempts to stop the fighting in the advice on thematic human rights issues.
region and aimed to implement an immediate Complaint Procedure: It allows individuals
ceasefire. and organisations to bring human rights
● The agreements relate to peace seeking violations to the attention of the Council.
settlements signed between Ukraine and ● Statement 3 is correct : The UNHRC holds
Russia and are named after the Belarusian regular sessions three times a year, in March,
capital where the agreements were first June, and September. The UNHRC can decide
settled. Minsk 1 - Signed in 2014, it was a 12- at any time to hold a special session to address
point ceasefire deal signed between Russia, human rights violations and emergencies, at
Ukraine , Organisation for Security and the request of one-third of the member states.
Cooperation in Europe (OSCE) and the pro-
Russia leaders from Donetsk and Luhansk in Q.66) Ans: C
Belarus. However, it could not stand. Minsk 2 - Exp:
Also known as Normandy Four, it was signed in ● Option 1 is not correct: Comprehensive
2015 between France, Germany, Ukraine, and Nuclear-Test-Ban Treaty, 1996 was
Russia after the failure of the Minsk negotiated at the Conference on

e
Agreement-I. Disarmament in Geneva and adopted by the

n
United Nations General Assembly in 1996. The

li
Q.65) Ans: B Treaty intends to ban all nuclear explosions -
Exp: everywhere, by everyone. It was opened for

n
● Statement 1 is not correct : The Human Rights signature in 1996 and since then 182 countries
o
Council is an inter-governmental body within have signed the Treaty, most recently Ghana
l.
the United Nations system responsible for has ratified the treaty in 2011. A
a

strengthening the promotion and protection comprehensive test ban has been defined as a
ri

of human rights around the world. The Council “zero yield” test ban that would prohibit
was created by the United Nations General supercritical hydro-nuclear tests but not sub-
te

Assembly in 2006. It replaced the former critical hydrodynamic nuclear tests.The Treaty
United Nations Commission on Human Rights. will enter into force after all 44 States listed in
a

The Office of the High Commissioner for Annex 2 to the Treaty will ratify it. These States
m

Human Rights (OHCHR) serves as the had nuclear facilities at the time the Treaty was
c

Secretariat of the Human Rights Council. negotiated and adopted. As of August 2011, 36
OHCHR is headquartered in Geneva, of these States have ratified the Treaty. Eight
s

Switzerland.It is made up of 47 United Nations States still need to do so: China, North Korea,
p

Member States which are elected by the UN Egypt, India, Iran, Israel, Pakistan and the
.u

General Assembly (UNGA). The UNGA takes United States. India, North Korea and Pakistan
into account the candidate States' contribution have not yet signed the Treaty.
w

to the promotion and protection of human ● Option 2 is not correct : India is one of the only
w

rights, as well as their voluntary pledges and five countries that either did not sign the NPT
commitments in this regard. or signed but withdrew, thus becoming part of
w

a list that includes Pakistan, Israel, North

DPP 2023 DAY 202 37


https://upscmaterial.online/
Download From - https://upscmaterial.online/

Contact us :info@onlyias.com

OnlyIAS Nothing Else Visit :dpp.onlyias.in


Contact : +91-7007 931 912

Korea, and South Sudan. India always Washington on 1st December 1959 for making
considered the NPT as discriminatory and had the Antarctic Continent a demilitarized zone to
refused to sign it. India has opposed the be preserved for scientific research only. The
international treaties aimed at non- twelve original signatories are Argentina,
proliferation since they were selectively Australia, Belgium, Chile, France, Japan, New
applicable to the non-nuclear powers and Zealand, Norway, South Africa, the Union of
legitimised the monopoly of the five nuclear Soviet Socialist Republics, the UK and the US.
weapons powers. It entered into force in 1961 and has since been
● Option 3 is correct : Chemical Weapons acceded by many other nations. Antarctica is
Convention: It is a multilateral treaty banning defined as all of the land and ice shelves south
chemical weapons and requiring their of 60°S latitude. Recently, an enormous
destruction within the stipulated time. iceberg 'A-76' has calved from the western side
Negotiations for the CWC began in 1980 at the of the Ronne Ice Shelf, lying in the Weddell Sea,
United Nations Conference on Disarmament. in Antarctica. Members: Currently it has 54
The convention was drafted in September parties. India became a member of this treaty
1992 and opened for signature in January in 1983.
1993. It became effective from April 1997. It ● Statement 3 is correct : Antarctica is to be
makes it mandatory to destroy old and demilitarised and used for peaceful purposes
abandoned chemical weapons. Members only. No military personnel can be stationed,
should also declare the riot-control agents and military weapons cannot be tested on the
(sometimes referred to as ‘tear gas’) in continent. Scientific research and

e
possession of them. Members: It has 192 state collaboration can be carried out, and the

n
parties and 165 signatories. India signed the findings are to be made freely available to

li
treaty in January 1993. signatories of the Treaty. Any territorial claims
over Antarctic territory are put on hold while

n
Q.67) Ans: C the Treaty remains in force. No nuclear testing
o
Exp: can be carried out in Antarctica or any
l.
● Statement 1 is correct : The Antarctic Treaty radioactive waste dumped on it.The Treaty
a

of 1959, which India is a signatory of, makes it only applies to ice shelves and land, not the
ri

mandatory for all the member countries to sea.


have provisions to prevent or check unlawful
te

activities at their research stations. The Q.68) Ans: B


Antarctic Treaty of 1959 provided that Exp:
a

Antarctica could only be used for the peaceful ● Option B is correct: Formally the Future
m

purpose of scientific investigation. It seeks to Investment Initiative (FII), it is widely being


c

preserve Antarctica forever by prohibiting described as “Davos in the desert”. The


military activity, nuclear explosions, and informal name derives from the World
s

radioactive waste disposals. The Antarctic Economic Forum’s annual meeting that is held
p

Treaty is applicable to the area south of the 60° in Davos, Switzerland, where world leaders
.u

South Latitude, including all ice shelves and discuss and shape agendas for pressing
islands. The Treaty entered into force in 1961 international issues. FII, too, brings together
w

and is therefore also referred to as the policymakers, investors and global experts,
w

Antarctic Treaty of 1961. who discuss the role of investment in driving


● Statement 2 is not correct : The Antarctic global prosperity and development. FII is an
w

Treaty was signed between 12 countries in initiative that was first undertaken by the Saudi

DPP 2023 DAY 202 38


https://upscmaterial.online/
Download From - https://upscmaterial.online/

Contact us :info@onlyias.com

OnlyIAS Nothing Else Visit :dpp.onlyias.in


Contact : +91-7007 931 912

Crown Prince Mohammad bin Salman in 2017 civilisational legacy, cultural diversity, political
to diversify the kingdom’s economy and system and past and ongoing contributions to
reduce its dependence on petroleum products. UN activities make
● India’s demand for a permanent seat in the
Q.69) Ans: D UNSC is completely rational.
Exp:
The Security Council was established by the UN Charter Q.70) Ans: B
in 1945. It is one of the six principal organs of the Exp:
United Nations. The council is headquartered at ● Statement 1 is correct: The NPT is an
NewYork. international treaty whose objective is to
● Statement 1 is correct: Its primary prevent the spread of nuclear weapons and
responsibility is to work to maintain weapons technology, to foster the peaceful
international peace and security uses of nuclear energy, and to further the goal
● Statement 2 is incorrect: The council has 15 of disarmament.
members: the five permanent members and ● Statement 2 is incorrect: The treaty was signed
ten non-permanent members elected for two- in 1968 and entered into force in 1970.
year terms. Presently, it has 190 member states. India is
● Statement 3 is correct: India, for the eighth not a member.
time, has entered the UNSC as a non- ● Statement 3 is correct: NPT requires countries
permanent member last year (2021) and will to give up any present or future plans to build
stay on the council for two years i.e 2021-22. nuclear weapons in return for access to

e
Each year, the General Assembly elects five peaceful uses of nuclear energy.

n
non-permanent members (out of ten in total)

li
for a two-year term. The ten non-permanent Extra Edge by OnlyIAS
seats are distributed on a regional basis ● NPT is the only binding commitment in a

n
● Statement 4 is incorrect : The five permanent multilateral treaty to the goal of disarmament
o
members are the United States, the Russian by the nuclear-weapon States
l.
Federation, France, China and the United ● India always considered the NPT as
a

Kingdom. discriminatory and had refused to sign it.


ri

● India has opposed the international treaties


Extra Edge by OnlyIAS aimed at non-proliferation since they were
te

India in the UNSC: selectively applicable to the non-nuclear


● India took active part in the formulation of the powers and legitimized the monopoly of the
a

Universal Declaration of Human Rights (UDHR) five nuclear weapons powers.


m

in 1947-48 and raised its voice passionately


c

against racial discrimination in South Africa. Q.71) Ans: D


● It has contributed extensively to the UN, Exp:
s

particularly for the maintenance of ● Nepal is an important neighbor of India and


p

international peace and security. occupies special significance in its foreign


.u

● India has taken part in 43 Peacekeeping policy because of the geographic, historical,
missions with a total contribution exceeding cultural and economic linkages/ties that span
w

160,000 troops and a significant number of centuries.


w

police personnel. ● Importance for India can be studied from two


● India’s population, territorial size, Gross different angles: a) their strategic importance
w

Domestic Product (GDP), economic potential, for India’s national security; and b) their place

DPP 2023 DAY 202 39


https://upscmaterial.online/
Download From - https://upscmaterial.online/

Contact us :info@onlyias.com

OnlyIAS Nothing Else Visit :dpp.onlyias.in


Contact : +91-7007 931 912

in India’s role perception in international Q.72) Ans: D


politics. Exp:
● Rivers originating in Nepal feed the perennial ● Statement 1 is correct : Indian Technical and
river systems of India in terms of ecology and Economic Cooperation Programme (ITEC) is a
hydropower potential. bilateral assistance programme run by the
Government of India. It is a demand-driven,
● Statement 1 is correct: Nepal being a response-oriented programme that focuses on
landlocked country, it is surrounded by India addressing the needs of developing countries
from three sides and one side is open towards through innovative technological cooperation
Tibet which has very limited vehicular access. between India and the partnering nation.
● India-Nepal has undertaken various ● Statement 2 is correct : The ITEC Programme is
connectivity programs to enhance people-to- fully funded by the Government of India.
people linkages and promote economic growth ● Statement 3 is correct : Division of
and development. Development Partnership Administration
● Statement 2 is correct: India from 2011, every (DPA) in the Ministry of External Affairs is the
year undertakes joint military exercise with nodal division for handling all capacity building
Nepal known as Surya Kiran. programmes.
● Statement 3 is correct: India Nepal share
various multilateral forums → India and Nepal Extra Edge by OnlyIAS
share multiple multilateral forums such as ● Assistance under ITEC covers the six areas of
BBIN (Bangladesh, Bhutan, India, and Nepal), training for civil and defence personnel,

e
BIMSTEC (Bay of Bengal Initiative for Multi projects and project related activities such as

n
Sectoral Technical and Economic Cooperation), consultancy services, study tours, donation of

li
Non Aligned Movement, and SAARC (South equipment, deputation of Indian experts in the
Asian Association for Regional Cooperation) partner nation and aid for disaster relief

n
etc ● Although ITEC is essentially a bilateral
o
● Statement 4 is correct: India is Nepal’s largest programme, its resources have also been used
l.
trade partner and the largest source of foreign for financing trilateral and regional
a

investments, besides providing transit for undertakings such as with the Economic
ri

almost the entire third country trade of Nepal Commission for Africa, UNIDO and G-77. In
Africa, Afro-Asian Rural Reconstruction
te

Extra Edge by OnlyIAS Organisation, COMESA, the African Union


Recently, the Union Cabinet has cleared a plan to build Commission and G-15 receive training and
a

a new bridge connecting India and Nepal over the project support under ITEC.
m

Mahakali river and link Dharchula in Uttarakhand with ● South Asian nations have traditionally been the
c

Nepal’s Dharchula. largest recipients of Indian aid both as part of


Mahakali River ITEC and otherwise.
s

● It is also known as Sharda river or Kali Ganga in ● Bhutan has traditionally been the largest
p

Uttarakhand. recipient of Indian aid. In Afghanistan and


.u

● It joins Ghagra river in Uttar Pradesh, which is Myanmar, India has focused on building
a tributary of Ganga. infrastructure such as roads, buildings and
w

● River Projects: Tanakpur hydro-electric hospitals besides providing training to their


w

project, Chameliya hydro-electric project, personnel. In Sri Lanka, Indian assistance has
Sharda Barrage. focused on rebuilding that country, especially
w

its Tamil majority areas in the north and east

DPP 2023 DAY 202 40


https://upscmaterial.online/
Download From - https://upscmaterial.online/

Contact us :info@onlyias.com

OnlyIAS Nothing Else Visit :dpp.onlyias.in


Contact : +91-7007 931 912

that were the worst affected, by fighting between them, in an independent manner.
during the Sri Lankan Civil War. Traditionally, India’s foreign policy towards
Israel and Palestine has been a hyphenated
Q.73) Ans: D foreign policy.However, hyphenating the ties
Exp: with Israel – linking them to ties with the
● Statement 1 and 3 are correct: The Palestinian Authority – essentially prevented
Turkmenistan-Afghanistan-Pakistan-India India from pursuing a pragmatic policy of what
Natural Gas Pipeline (TAPI) Project is a was in India’s best interests.India in the recent
proposed 1,814km trans-country natural gas year has been following a dehyphenation
pipeline running across four countries. policy between Israel and Palestine. It means
○ This pipeline is also known as “Peace India’s relationship with Israel would stand on
Pipeline”. its own merits, independent and separate from
○ These countries are – Turkmenistan, India’s relationship with the Palestinians.
Afghanistan, Pakistan and India.
● Statement 2 is correct: The TAPI project is Q.75) Ans: A
being funded by the Asian Development Bank Exp:
(ADB), which is also acting as transaction ● Statement 1 is correct: The OSCE is a security
adviser for the development. organization with 57 member states in North
America, Europe and Asia. It deals with a wide
range of security-related concerns,including
Extra Edge by OnlyIAS arms control, nonproliferation,
● The pipeline will transport natural gas from the counterterrorism, democratization, and

e
Galkynysh Gas Field in Turkmenistan through human rights.

n
Afghanistan into Pakistan and then to India. ● Statement 2 is correct and Statement 3 is
incorrect: With 57 participating States(India is

li
● The pipeline will enter India through the Indian
town of Fazilka (near the Indo-Pak border). not among the 57 states) in North America,

n
Europe and Asia, the Organization for Security
o
and Co-operation in Europe – is the world’s
largest regional security organization.
l.
a
ri

Q.76) Ans: B
te

Exp:
● Option B is correct: Recently India and Israel
along with the US and UAE formed the ‘New
a

Quad’ or the ‘Middle-Eastern Quad’ to work on


m

furthering the economic and political ties


between the four countries. Member countries
c

of New Quad grouping are - India, the US,


s

Israel and the UAE. India’s involvement in this


new grouping reflects a shift in its foregin
p

policy.
.u

Q.74) Ans: B
w

Exp:
● Option B is correct: In international politics,
w

Q.77) Ans: D
de-hyphenation means dealing with two Exp:
w

countries, having an adversarial relationship

DPP 2023 DAY 202 41


https://upscmaterial.online/
Download From - https://upscmaterial.online/

Contact us :info@onlyias.com

OnlyIAS Nothing Else Visit :dpp.onlyias.in


Contact : +91-7007 931 912

● Statement 1 is not correct : Ashgabat and to expand international cooperation in


Agreement is a multimodal transport the Asia-Pacific region.
agreement. It aims at easing the ● Statement 2 is not correct : This forum is
transportation of goods by developing the sponsored by the organising committee
international transport and transit corridor appointed by Roscongress, an association of
between Central Asia and the Persian Gulf. the Russian Government, which also sponsors
This agreement led to the construction of the other international forums, such as St.
railway lines, harmonising multiple important Petersburg International Economic Forum.
projects. It further developed the connectivity This forum serves as a platform for the
of Central Asia and Eurasia with the Gulf and discussion of key issues in the world economy,
INSTC. In 2014, the agreement was given regional integration, and the development of
additional assistance by signing the new industrial and technological sectors, as
Memorandum of Understanding (MoU), which well as of the global challenges facing Russia
explained various fields such as legal and other nations.
procedure and judicial, infrastructure, etc., ● Far East : It is the easternmost part of Russia.
considering the trade and transit agreement. It borders two oceans, the Pacific and the
● Statement 2 is not correct : The agreement Arctic, and five countries (China, Japan,
was originally signed by Iran, Oman, Qatar, Mongolia, the United States and the DPRK).
Turkmenistan and Uzbekistan in 2011. Qatar The Far Eastern Federal District covers more
subsequently withdrew from the agreement in than a third of the country’s territory. It is rich
2013, the same year Kazakhstan applied for in natural resources like diamonds, stannary,

e
membership, which was eventually approved borax materials, 50 gold, tungsten, and fish

n
in 2015. Pakistan has also joined the and seafood. About 1/3 of all coal reserves and

li
Agreement from November 2016. India hydro-engineering resources of the country
formally joined in February 2018. China is not are here. Forests of the region comprise about

n
a member of Ashgabat agreement. The 30% of the total forest area of Russia.
o
Ashgabat agreement is a multimodal transport
l.
agreement between the governments of
a

Kazakhstan, Uzbekistan, Turkmenistan, Iran, Q.79) Ans: D


ri

India, Pakistan, and Oman for creating an Exp:


international transport and transit corridor ● Option D is correct : During the Indian Prime
te

facilitating transportation of goods between Minister's Visit to Denmark, India and


Central Asia and the Persian Gulf. The Denmark agreed to further strengthen the
a

agreement came into force in April 2016. Green Strategic Partnership with a focus on
m

Ashgabat in Turkmenistan is the depository green hydrogen, renewable energy and


c

state for the agreement wastewater management. Further, India


conveyed its acceptance of the Danish
s

Q.78) Ans: C invitation to join the International Center for


p

Exp: Antimicrobial Resistance Solutions (ICARS) as a


.u

● Statement 1 is not correct : Eastern Economic Mission Partner. The Danish Prime Minister
Forum is an international forum organised confirmed Danish accession to the Global
w

each year in Vladivostok, Russia. It was first Digital Health Partnership on India’s invitation
w

held in September 2015, at Far Eastern Federal to improve public health and well-being
University in Vladivostok, to support the through evidence-based digital technologies.
w

economic development of Russia’s Far East

DPP 2023 DAY 202 42


https://upscmaterial.online/
Download From - https://upscmaterial.online/

Contact us :info@onlyias.com

OnlyIAS Nothing Else Visit :dpp.onlyias.in


Contact : +91-7007 931 912

● Green Strategic Partnership: The Partnership some of India’s neighbours and has pushed
will focus on expanding economic ties, green those countries into debt traps.
growth, and cooperation on global challenges ● Statement 2 is correct : India and the United
such as climate change. Green growth is a Kingdom have launched their Global
term to describe a path of economic growth Innovation Partnership (GIP) as both
that uses natural resources in a sustainable countries look to bolster their innovation
manner. Danish companies with niche ecosystem to benefit the start-ups. GIP is
technologies and expertise have offered to expected to provide a model for utilising the
help India in meeting its air pollution control Trilateral Development Corporation (TDC)
targets, including in the key area of tackling the Fund for projects with other countries like
problem of burning crop stubble. Other key Germany, Japan, the European Union, France,
points under the partnership include dealing etc that are willing to work with India on the
with the Covid-19 pandemic and cooperation field of development and innovation. The GIP
in water efficiency and water loss. The creation will be co-financed by India and the UK.
of India-Denmark energy parks in areas with
large numbers of Danish firms and an India- Q.81) Ans: D
Denmark skill institute to train Indian Exp:
manpower has been proposed. The Green ● Statement 1 is not correct : The IMCG has
Strategic Partnership will build on an existing been set up as a high-level mechanism
Joint Commission for Cooperation and towards mainstreaming of India’s
existing joint working groups. ‘Neighbourhood First’ policy vision that

e
sought to develop better relations with the

n
country’s neighbours. IMCG is supported by

li
inter-ministerial Joint Task Forces (JTFs)
Q.80) Ans: B convened by joint secretaries in the external

n
Exp: affairs ministry. The IMCG provided a
● Statement 1 is not correct : The TDC fund has
o
comprehensive direction with a whole-of-
l.
been recently launched by India to involve government approach to promote better
a

private sectors with the support of the state connectivity, stronger interlinkages and
ri

for investments in the Indo-Pacific region. This greater people-to-people connect with the
region comprises the Indian Ocean, the central neighbours. The focus of the meeting was
te

and western Pacific Ocean, and the seas that construction of border infrastructure that
connect the two around Indonesia and would facilitate greater trade with neighbours
a

beyond. like Nepal; special needs of countries such as


m

● Aim of the TDC Fund : India has aims of using Bhutan and Maldives in terms of supply of
the TDC Fund to contribute to the GIP that will
c

essential commodities; opening rail


help in globalising the Indian innovations and connectivity with Bangladesh; Humanitarian
s

start-ups with help like developmental capital assistance to Afghanistan and Myanmar; and
p

investment, grants, and tech assistance. Fisheries issue with Sri Lanka.
.u

Investments through this fund will be ● Statement 2 is not correct : Recently, the first
increased in those places where Indian meeting of the Inter-Ministerial Coordination
w

companies are already investing. The TDC Fund Group (IMCG) at Secretary level was
w

will be providing an alternative to the convened by India’s Foreign Secretary.IMCG


development partnership model of China that will further improve institutional coordination
w

has deepened the influence of China over across government and provide

DPP 2023 DAY 202 43


https://upscmaterial.online/
Download From - https://upscmaterial.online/

Contact us :info@onlyias.com

OnlyIAS Nothing Else Visit :dpp.onlyias.in


Contact : +91-7007 931 912

comprehensive direction to this whole-of- the Chairmanship in June 2021 for a two-year
government approach to India’s relations with tenure.
its neighbouring countries.The timing of the
initiative is significant as it comes within days Q.83) Ans: A
of the massive protests over the economic Exp:
situation in Sri Lanka. The IMCG could also be ● Statement 1 is not correct : One Country Two
convened to help avoid tensions at the land Systems approach : As per the policy, the
borders with Pakistan, Nepal and Bangladesh. Hong Kong and Macau Special Administrative
Regions, both former colonies, can have
different economic and political systems from
Q.82) Ans: A that of mainland China, while being part of
Exp: the People’s Republic of China. It was
● Statement 1 is not correct : The Indian Ocean proposed by Deng Xiaoping with an aim to
Naval Symposium is a voluntary and inclusive unify China and Taiwan. On December 19,
initiative that brings together navies of Indian 1984, China and the U.K. signed the Sino-
Ocean Region littoral states to increase British Joint Declaration in Beijing, which set
maritime co-operation and enhance regional the terms for the autonomy and the legal,
security.It also serves to develop an effective economic and governmental systems for Hong
response mechanism and humanitarian Kong post 1997. Similarly, on March 26, 1987,
assistance. Recently, the 7th edition of Indian China and Portugal signed the Joint Declaration
Ocean Naval Symposium (IONS) was hosted by on the Question of Macau in which China made

e
the French Navy at Réunion island. It is a similar promises for the region of Macau after

n
biennial event which was conceived by the it was handed over to Beijing.
Indian Navy in 2008.

li
● Statement 2 is correct : Hong Kong returned to
● Statement 2 is correct : IONS includes 24 Chinese control on July 1, 1997, and Macau’s

n
member nations that touch or lie within the sovereignty was transferred on December 20,
o
Indian Ocean Region (IOR), and 8 observer 1999. Both regions became Special
l.
nations. The members have been Administrative Regions of China. The regions
a

geographically grouped into the following four would have their own currencies, economic
ri

sub-regions: South Asian Littorals: Bangladesh, and legal systems, but defence and diplomacy
India, Maldives, Pakistan, Seychelles, Sri Lanka would be decided by Beijing. Their mini-
te

and United Kingdom (British Indian Ocean Constitutions would remain valid for 50 years
Territory) West Asian Littorals: Iran, Oman, — till 2047 for Hong Kong and 2049 for
a

Saudi Arabia and United Arab Emirates East Macau. It is unclear what will happen after
m

African Littorals: France (Reunion), Kenya, this term.


Mauritius, Mozambique, South Africa, and
c

Tanzania. South East Asian and Australian Q.84) Ans: D


s

Littorals: Australia, Indonesia, Malaysia, Exp:


p

Myanmar, Singapore, Thailand and Timor- ● Statement 1 is not correct : I2U2 stands for
.u

Leste. India, Israel, the UAE, and the US, and was
● Statement 3 is not correct : The chairmanship also referred to as the ‘West Asian Quad’.
w

of IONS has been held by India (2008-10), UAE I2U2 was initially formed in 2021 following
w

(2010-12), South Africa (2012-14), Australia the Abraham Accords, to deal with issues
(2014-16), Bangladesh (2016-18) and Islamic concerning maritime security, infrastructure
w

Republic of Iran (2018-21). France assumed and transport. The Abraham Accord is the first

DPP 2023 DAY 202 44


https://upscmaterial.online/
Download From - https://upscmaterial.online/

Contact us :info@onlyias.com

OnlyIAS Nothing Else Visit :dpp.onlyias.in


Contact : +91-7007 931 912

Arab-Israeli peace deal in 26 years mediated by scenario. As the name suggests, competitive
the USA.Its stated aim is to discuss “common markets that are imperfect in nature.
areas of mutual interest, to strengthen the ● Statement 2 is correct: Monopoly is an
economic partnership in trade and investment example of imperfect competition. Such
in our respective regions and beyond”. Six imperfect competition will cause market
areas of cooperation have been identified by inefficiency when it happens, resulting in
the countries mutually, and the aim is to market failure. Imperfect competition can be
encourage joint investments in water, energy, found in the following types of market
transportation, space, health, and food structures: monopolies, oligopolies,
security. monopolistic competition, monopsonies, and
● Statement 2 is not correct : The Abraham oligopsonies.
Accord between Israel, the United Arab
Emirates and is mediated by the USA. It is the Q.87) Ans: B
first Arab-Israeli peace deal in 26 years. Exp:
Previous Agreements: There were the only two ● Option B is correct: Windfall gain (or windfall
peace deals between Israel and the Arab States profit) is an unexpected gain in income which
in more than a quarter of a century. Egypt was could be due to winning a lottery, unforeseen
the first Arab State to sign a peace deal with inheritance or shortage of supply. Windfall
Israel in 1979. Jordan signed a peace pact in gains are transitory in nature. For instance,
1994. when real estate property prices rise
dramatically, the owner can make a substantial

e
Q.85) Ans: C amount of profit by selling property. This

n
Exp: sudden and unexpected rise in income is called

li
windfall profit. Many countries define proper
● Statement 1 is correct: Domestic institutional laws to tax windfall profits.

n
investors are those institutional investors o
which undertake investment in securities and Q.88) Ans: C
l.
other financial assets of the country they are Exp:
a

based in. DIIs include banks, insurance ● Option C is correct: Tulip mania was a period
ri

companies, mutual fund houses, etc. when tulips were recently introduced and
● Statement 2 is correct: These investment bought in large quantities by many people. This
te

decisions are influenced by various domestic caused tulip prices to shoot up. They were sold
economic as well as political trends. In addition at prices higher than skilled workers' income.
a

to the foreign institutional investors, the After reaching a peak, tulip prices crashed,
m

domestic institutional investors also affect the leaving tulip holders bankrupt. It was the first
c

net investment flows into the economy. major economic bubble.


s

Q.86) Ans: B
p

Exp: Q.89) Ans: B


.u

● Statement 1 is incorrect: Imperfect Exp:


competition is a competitive market situation ● Statement 1 is incorrect: A black swan is an
w

where there are many sellers, but they are unpredictable event that is beyond what is
w

selling heterogeneous (dissimilar) goods as normally expected of a situation and has


opposed to the perfect competitive market potentially severe consequences.
w

DPP 2023 DAY 202 45


https://upscmaterial.online/
Download From - https://upscmaterial.online/

Contact us :info@onlyias.com

OnlyIAS Nothing Else Visit :dpp.onlyias.in


Contact : +91-7007 931 912

● Statement 2 is correct: Black swan events are cloning, and be used to make payments and
characterized by their extreme rarity, severe transact with other bank accounts.
impact, and the widespread insistence they
were obvious in hindsight. Q.93) Ans: B
Exp:
Q.90) Ans: B ● Statement 1 is incorrect: The paradox of thrift
Exp: refers to a situation in which people tend to
● Statement 1 is incorrect: Dovish support the save more money, which essentially leads to a
idea of low-interest rates since they believe fall in aggregate demand and hence in
that it encourages economic growth. In order
economic growth.In other words, when
to keep inflation in check, the Hawkish stance
favours high-interest rates. everyone increases their saving-income
● Statement 2 is correct: Hawkish Stance are proportion, MPS, then aggregate demand falls
primarily interested in high-interest rates as as consumption reduces.
they relate to Fiscal policy. They are generally ● Statement 2 is correct: The paradox of thrift
not concerned with economic growth but postulated by the British Keynesian economist,
support an economy operating at a level below John Maynard Keynes. His theory was based
its full-employment equilibrium.
upon Keynesian economic theory that
productivity is driven by aggregate demand.
Q.91) Ans: A ● Statement 3 is correct: When consumers save
Exp: (rather than spend), it harms the businesses
● Statement 1 is correct: Calibrated tightening that serve the needs and wants of consumers.

e
means during the current rate cycle, a cut in This leads to a decrease in the level of
the repo rate is off the table. However, the rate

n
employment and income and reduces total
hike will happen in a calibrated manner.

li
savings in the economy.
● Statement 2 is correct: Calibrated Tightening

n
means the central bank may not go for a rate
increase in every policy meeting but the overall Q.94) Ans: C o
policy stance is tilted towards a rate hike. Exp:
l.
● Statement 3 is incorrect: Calibrated Tightening ● Both Statements 1 and 2 are incorrect: Catch
a

can happen outside the policy meetings as well up effect, alternatively called the theory of
if the situation warrants.
ri

convergence, states that poor or developing


economies grow faster compared to
te

developed economies (economies with a


Q.92) Ans: C
higher per capita income and gradually reach
a

Exp:
similar high levels of per capita income). Thus,
m

● Option C is correct: Credit card cloning or


all economies, over time, may converge in
skimming is the illegal act of making
c

terms of income per head.


unauthorised copies of credit or debit cards.
s

This enables criminals to use them for


p

payments, effectively stealing the cardholder’s Q.95) Ans: B


.u

money and/or putting the cardholder in debt. Exp:


Once the device picks up the data, it can be ● Statement 1 is correct: An indifference curve is
w

used to gain unauthorised access to the user’s a graph showing a combination of two goods
banking records. The stolen information can be
w

that give the consumer equal satisfaction and


coded onto a new card, a process called utility. Each point on an indifference curve
w

indicates that a consumer is indifferent

DPP 2023 DAY 202 46


https://upscmaterial.online/
Download From - https://upscmaterial.online/

Contact us :info@onlyias.com

OnlyIAS Nothing Else Visit :dpp.onlyias.in


Contact : +91-7007 931 912

between the two and all points give him the ● Statement 1 is correct: Recessionary gap is
same utility. also termed as contractionary gap. An
● Statement 2 is incorrect: The indifference economy doesn't necessarily operate at the
curve is drawn as a downward sloping convex full employment level. So the difference that
to the origin. The graph shows a combination exists between the potential full employment
of two goods that the consumer consumes. equilibrium and the actual ones is the
● Statement 3 is correct: Higher indifference recessionary gap.
curve represents a higher level of satisfaction. ● Statement 2 is incorrect: Recessionary Gap is a
Higher indifference curve represents larger situation wherein the real GDP is lower than
bundles of goods i.e. bundles which contain the potential GDP at the full employment level.
more of both or more of at least one. It is The economy operates below the full
assumed that consumer’s preferences are employment level in a recessionary gap.
monotonic i.e. he always prefers larger
bundles as it gives him higher satisfaction. Q.99) Ans: B
Exp:
Q.96) Ans: A ● Statement 1 is incorrect: Stimulus package is a
Exp: package of tax rebates and incentives used by
● Option A is correct: The contagion effect the governments of various countries to
explains the possibility of spread of economic stimulate the economy and save their country
crisis or boom across countries or regions. This from a financial crisis.
phenomenon may occur both at a domestic ● Statement 2 is correct: The idea behind a

e
level as well as at an international level. The stimulus package is to provide tax rebates and

n
failure of Lehman Brothers in the United States boost spending, as spending increases

li
is an example of a domestic contagion. demand, which leads to an increase in
employment rate which in turn increases

n
Q.97) Ans: A income and hence boosts spending. This cycle
o
Exp: continues until the economy recovers from
l.
● Option A is correct: Pareto efficiency or Pareto collapse. One such stimulus package was used
a

optimality is a situation where no individual or by the United States in 2008 during the time of
ri

preference criterion can be made better off the global recession, which was aimed at
without making at least one individual or increasing employment and recovery of the US
te

preference criterion worse off. Pareto economy.


originally used the word "optimal" for the
a

concept, but as it describes a situation where a Q.100) Ans: B


m

limited number of people will be made better Exp:


c

off under finite resources, and it does not take ● Statement 1 is incorrect: Monetary policy
equality or social well-being into account, it is refers to the policy of the central bank with
s

in effect a definition of and better captured by regard to the use of monetary instruments
p

"efficiency". under its control to achieve the goals specified


.u

in the Act. The primary objective of the RBI’s


monetary policy is to maintain price stability
w

while keeping in mind the objective of growth.


w

Q.98) Ans: A ● Statement 2 is correct: Composition: Section


Exp: 45ZB says the MPC shall consist of 6 members:
w

1. RBI Governor as its ex officio chairperson,

DPP 2023 DAY 202 47


https://upscmaterial.online/
Download From - https://upscmaterial.online/

Contact us :info@onlyias.com

OnlyIAS Nothing Else Visit :dpp.onlyias.in


Contact : +91-7007 931 912

2. Deputy Governor in charge of monetary policy,


3. An officer of the Bank to be nominated by the
Central Board,
4. Three persons to be appointed by the central
government.
a. This category of appointments must be
from “persons of ability, integrity and
standing, having knowledge and
experience in the field of economics or
banking or finance or monetary
policy”.

e
n
li
n
o
l.
a
ri
te
a
m
c
s
p
.u
w
w
w

DPP 2023 DAY 202 48


https://upscmaterial.online/

You might also like